Sunteți pe pagina 1din 45

Transition Element_1

Address: MHV Plaza, Krishi FarmRoad, Kela Nagar Aligarh, Phone: 0571-6543330/1
TRANSITION ELEMENTS
1. SYLLABUS
Definition, general characteristics, oxidation states and the color (excluding the details of electronic
transitions) and calculation of spin-only magnetic moment; Coordination compounds : Nomenclature
of mononuclear coordination compounds, cis-transitions isomerism, hybridization and geometries of
mononuclear coordination compounds(linear, tetrahedral, square planar and octahedral).
2. INTRODUCTION
The elements lying between s and p-block elements of the periodic table are collectively known as
transition or transitional elements. These elements either in their atomic state or in any of their
common oxidation state have partly filled (n1) dorbitals of (n1)th main shell. In these elements the
differentiating electron enters (n1)d orbitals of (n1)th main shell and as such these are called d-
block elements.
The name transition is given to the elements on the basis of their position in the P.T and their
properties i.e. they occupy a position between the highly electropositive elements on the left and the
electronegative elements on the right. Their properties are also intermediate of the s- and p-block
elements
In the d-block, electrons are added to the penultimate shell, expanding it from 8 to 18 electrons. All the
d-block elements are classified into four series i.e. 3d, 4d, 5d and 6d series corresponding to the filling
of 3d, 4d, 5d and 6d orbitals of (n1)th main shell. Each of 3d, 4d and 5d series has ten elements while
6d series has at present only one element viz.
90
Ac whose valence shell configuration is
1 2
6d 7s .
3. ELECTRONIC CONFIGURATION AND IRREGULARITIES
The valence shell configurations of these elements can be represented by
, ,
( )
1 10 012
n 1d ns . The
observed valence shell configuration of these elements differ from their predicted configuration. The
irregularities in the observed configurations of
5 1 10 1 10 0 10 0
Cr(3d 4s ), Cu(3d 4s ), Mo(4d 5s ), pd(4d 5s )
10 1
Ag(4d 5s ) and
10 1
Au(5d 6s ) are explained on the basis of the concept that half-filled and
completely filled d-orbitals are relatively more stable than other d-orbitals. Only this factor is not
sufficient to explain the irregularities in observed electronic configuration. The net effect of all the
forces, comprising (i) nuclear electronic attraction (ii) shielding of one electron by others from other
nuclear charge (iii) inter electronic repulsion and (iv) exchange forces determines the stability of the
electronic configuration. It is not easy to explain why unlike
5 1
Cr(3d 4s ) and
5 1
Mo(4d 5S ) should
have the idealized electronic configuration
14 4 2
(4f 5d 6s ).
The configurations clearly show that strictly, According to the definition of d-block elements; Cu, Ag
and Au should be excluded from d-block elements, since these elements both in their atomic state (with
configuration (n1)
10 1
d ns ) and in their +1 oxidation state (with configuration (n1)
10
d ), do not have
partly filled (n1) d-orbitals. Similarly Zn, Cd and Hg which both in their atomic state [( ) ]
10 2
n 1d ns
and in +2 oxidation state
10
[(n1)d ] do not contain partly filled [(n1)d] orbitals, should also be
excluded from d-block elements. Similarly is the case with Pd atom with configuration
10 0
4d 5s . yet,
in order to maintain a rational classification of elements, these elements (viz cu, Ag, Au, Zn, Cd, Hg
and Pd) are also generally studied with d-block element.
Question 1: The mercurous ion is written as
2
2
Hg
+
while the cuprous ion is written as Cu
+
.
Solution:
14 10 2
80 54
Hg :[Xe] 4f 5d 6s
14 10 1
Hg :[Xe]4f 5d 6s
+
Transition Element_2
Address: MHV Plaza, Krishi FarmRoad, Kela Nagar Aligarh, Phone: 0571-6543330/1
In
I
Hg , there is one unpaired electron and we expect (mercurous) salt to be
paramagnetic. But magnetic moment of mercurous salt is zero indicating that it is
diamagnetic which is only possible when 6s electron has been used by two Hg atoms in
bonding. Hence mercurous ion is
2
2
Hg
+
.
10 1
29
Cu :[Ar]3d 4s
10
Cu :[Ar]3d
+
I
Cu (cuprous) salt is diamagnetic by experiment and thus it is not dimeric and is thus
Cu
+
.
4. PHYSIOCHEMICAL PROPERTIES
The difference in the trends in the properties of d-block elements from those of s- and p-block elements
arises from a basic difference in their electronic configuration while in the building up of elements
from Li to F, the electrons are added to the outer most shell but in the case of transition metals, the
electrons are added to inner (n1)d sub-shell. Although, the horizontal similarity amongst the d-block
elements is well marked, yet the chemistry of the elements of first transition series differ considerably
from that of the elements of the second and third transition series which are more similar to each other.
Some properties of d-block elemenets
Property Scandium Titanium Vanadium Chromium Manganese Iron Cobalt Nickel Coper Zinc
Symbol Sc Ti V Cr Mn Fe Co Ni Cu Zn
Atomic No. 21 22 23 24 25 26 27 28 29 30
Atomic weight 44.956 47.90 50.942 51.996 54.938 55.847 58.933 58.710 63.54 65.37
Metallic radius
(pm)
164 147 135 130 135 126 125 125 128 137
Ionic radius
(pm)
81
(3+)
76, 68
(3+) (4+)
74, 60
(3+) (4+)
84, 69
(2+) (3+)
80, 66
(2+) (3+)
76, 64
(2+)
(3+)
74, 63
(2+)(3+)
72, 62
(2+)(3+)
96, 69
(1+)(2+)
74
(2+
Covalent radius
(pm)
144 132 122 118 117 117 116 115 117 125
Boiling point (K) 3000 3533 3673 2753 2370 3273 3173 3005 2868 1180
Melting point (K) 1812 1948 2173 2163 1517 1808 1768 1726 1356 692
Density
3 3
10 kgm
3.0 4.5 6.11 7.2 7.44 7.86 8.86 8.90 8.92 7.13
Electronegativity
(A/R)
1.2 1.3 1.45 1.55 1.6 1.65 1.7 1.75 1.75 1.65
Ionisation 1
st
633 659 650 653 717 762 759 736 745 906
Energy 2nd 1235 1309 1414 1591 1509 1561 1644 1751 1958 1732
(kJ
1
mol ) 3rd
2388 2648 2866 2992 3259 2958 3230 3391 3556 3828
Electrode (III) (III) (IV) (II) (III) (II) (III) (II) (III) (II)(III) (II) (III) (II) (I) (II) (II)
Potential (V)
+
2.1 1.2
1.63
1.2
0.86
0.91
0.74
1.18
0.28
0.44
0.04
0.28
+0.4
0.25 +0.52
+0.34
0.76
4.1 ATOMIC (COVALENT) AND IONIC RADII, ATOMIC VOLUME AND DENSITY
There is a gradual decrease in atomic radius and ionic radius on moving from left to right because,
additional positive charges are placed on the nucleus and correspondingly electrons are added to the
(n1)d orbitals. As the electrons in the d-orbitals shield the ns electrons and also themselves from the
nuclear charge incompletely, effective nuclear charge felt by them increases and hence a contraction in
size occurs. The atomic radii for the elements from Cr to Cu are very close to one another because
Transition Element_3
Address: MHV Plaza, Krishi FarmRoad, Kela Nagar Aligarh, Phone: 0571-6543330/1
simultaneous addition of electron to 3d-level exercise the reverse effect by screening the outer 4s-
electrons from the inward pull of the nucleus. As a result of these two opposing effects, the atomic
radii do not alter much on moving from Cr to Cu.
As we move from alkali metals to transition elements, radii decreases steeply but within transition
elements this rate of decrease is less due to an increase in interelectronic repulsion.
The ionic radii of M
2+
and M
3+
ions follow the same trends as their atomic radii. The radii of M
2+
ions,
although somewhat smaller than that of Ca
2+
ion (0.99) are comparable with it. Thus MO type oxides
of the transition element should be similar to CaO in many ways, although somewhat less basic and
less soluble in water. Similarly the hydration energies of M
2+
ions should be similar to but somewhat
greater than that of Ca
2+
ion. This is borne out by facts, since the hydration energy of Ca
2+
ion is 395
kcal and the observed values of hydration energies for the elements Ti
2+
Cu
2+
are between 446 kcal
and 597 kcal.
The radii of M
3+
ions are slightly greater than that of Ga
3+
ion (0.62). Thus M
2
O
3
oxides of transition
elements should be similar to but slightly less acidic (more basic) than Ga
2
O
3
and the hydration
energies of M
3+
ions should be less than 1124 kcal which is the hydration energy of Ga
3+
ion. The
observed values of hydration energies for the series Sc
3+
Fe
3+
are between 947 kcal and 1072 kcal.
Atomic volumes follows the same trend (decreasing) as the atomic size. While there is a general trend
of increasing densityacross transition series because smaller the size, the higher is the density.
4.2 METALLIC CHARACTER
All the transition elements are metals, since the number of electrons in the outer-most shell is very
small, being equal to 2. They are hard, malleable and ductile. They exhibit all the three types of
structures: face centred cubic (fcc), hexagonal close packed (hcp) and body centred cubic (bcc).
Metals of VIII and IB groups are softer and more ductile than other metals. These metals are good
conductors of heat and electricity
4.3 MELTING AND BOILING POINTS
The m.p. and b.p. of transition elements are usually high in comparison with s- and p block elements.
The m.p of elements (i.e.-transition elements) depend upon the strength of metallic bond which
increases with the availability of the unpaired d-electrons to participate in the bonding by
delocalization. In first transition series, there is a sharp decrease of m.p. after the middle (Mn having 5
unpaired electrons) in the series due to electron pairing. Thesoftness and low m.p. of Zn, Cd and Hg
(liquid) is due to pairing of all electrons. Periodic trends in the b.p. are similar to those in m.p.
4.4 IONISATION POTENTIAL
The first ionisation potentials of transitional elements lie between the values of those of s- and p-block
elements. The first ionisation potentials of all the transition elements lie between 5 to 10 electron volts.
In case of transition elements the addition of the extra electron in the (n-1) d level provides a screening
effect which shields the outer ns electrons from the inward pull of positive nucleus on the outer ns
electrons. Thus the effects of the increasing nuclear charge and the shielding effect created due to the
expansion of (n-1)d orbital oppose each other. On account of these counter effects, the ionisation
potentials increase rather slowly on moving in a period of the first transition series.
a) First ionisation potentials : It is evident that the values for the first four 3d block elements (Sc,
Ti, V and Cr) differ only slightly from one another. Similarly the values for Fe, Co, Ni and Cu
also are fairly close to one another. The value of IE
I
for Zn is considerably higher. This is due to
the extra-stability of 3d
10
level which is completely filled in Zn atom.
Transition Element_4
Address: MHV Plaza, Krishi FarmRoad, Kela Nagar Aligarh, Phone: 0571-6543330/1
b) Second ionisation potentials : The second ionisation potentials are seen to increase more or less
regularly with the increase of atomic number. The value of IE
II
for Cr and Cu are higher than
those of their neighbours. This is due to the fact that the electronic configurations of Cr
+
and Cu
+
ions have extra stable 3d
5
and 3d
10
levels.
There is a sudden fall in the values of ionisation potentials in going from II B (Zn-group elements)
to IIIA sub-group. This sudden fall is explained on the basis that in case of IIIA group elements
the electron to be removed is from a 4p-orbital which is incompletely filled, while in case of the II
B group elements, the electron to be removed is from 4s-orbital which is completely filled. Thus
more energy will be required to remove an electron from a filled 4s-orbital in comparison to that
used to remove an electron from a 4p-orbital which is incompletely filled.
c) Electropositive character of transitional elements as compared to that of alkali metals and
alkaline earth metals : The values of first ionisation potentials of transition elements in most
cases lie between those of s-and p-block elements. Thus the transition elements are less
electropositive than the elements of I A and II A groups. Thus, although the transitional elements
do formionic compounds, yet they do not form ionic compounds so readily as the alkali and
alkaline earth metals do. Also, unlike the alkali and alkaline earth metals, the transitional elements
also have a tendency to form the covalent compounds under certain conditions. Generally the
compounds in which the transition elements show a smaller valency are ionic, while those in which
a higher valency is exhibited are covalent in character.
4.5 ELECTRONEGATIVITY
Transition element have fairly low values of electronegativity. It increases from Sc to Cu with a fall at
Mn and Zn. However, this increase in electronegativity is much slower because the additional electron
is being added to an inner shell which provides relatively good shielding to the outer electrons from the
nucleus. The increasing electronegativity from Sc to Cu means that the elements become slightly less
metallic and this is reflected in the increasing positive electrode potentials of their ions
2
M
+
and
3
M
+
.
4.6 ELECTRODE POTENTIAL
The potential difference set up in a 1M solution of metal ions at 298 K is called standard electrode
potential against hydrogen electrode as reference.
Electrode potential is a measure of the electro positive character and it decreases along first transition
series (except Cu which has negative value) and can react with acids producing hydrogen.
4.7 OXIDATION STATES
Transition elements exhibit a wide range of oxidation states differing usually by units of one. This is
due to the fact that (n1) d electrons may get involved along with ns electrons in bonding, as electrons
in (n1) d orbitals are in an energy state comparable to ns electrons. There exists a general trend of
lesser no of oxidation states at each end of the series and a higher no in the middle. The lesser no of
oxidation states in the beginning of the series can be due to the presence of too few electrons to lose or
share towards the end of series it can be ascribed to the presence of too many electrons and thus fewer
empty orbitals to share electrons with the ligands. Another feature is the reduced tendency of higher
oxidation states towards the end of the series. This could be due to steady increase in the effective
nuclear charge along the series, thus pulling the d-orbitals into the electron core and not making them
readily available for bonding.
i) Minimum oxidation state. All the transition elements with the exception of Cr, cu, Ag, Au and
Hgwhich have a minimum oxidation state of +1 exhibit a minimum oxidation stateof +2. In most
cases this +2 oxidation state arises due to the loss of two s-electrons.
ii) Maximum oxidation state. Each of the elements in groups III B to VII B can show the maximum
oxidation state equal to its group number. Thus, Cr in group VIB shows a maximum oxidation
state of +6 in
2
2 7
Cr O ion. Most of the elements in VIII group show a maximum oxidation sate
equal to +6. However, Ru and Os have a maximum oxidation state equal to +8 which is the
highest oxidation state shown by any element.
Transition Element_5
Address: MHV Plaza, Krishi FarmRoad, Kela Nagar Aligarh, Phone: 0571-6543330/1
iii) Relative stability of various oxidation states. The relative stabilities of various oxidation states
of 3d- series elements can be correlated with the extra stability of
0 5
3d ,3d and
10
3d
configurations to some extent. Thus
4
Ti
+ 0
(3d ) is more stable than
3 1
Ti (3d )
+
and similarly
2 5
Mn (3d )
+
is more stable than
4 4
Mn (3d )
+
. It may, however, be pointed out that such a
generalization for the relative stability of various oxidation states of 4d- and 5d- series elements is
often rather difficult to realise.
The higher oxidation states of 4d and 5d series elements are generally more stable than those of
the elements of 3d series, e.g., Mo, Te (4d-series elements) and W, Re (5d-series elements) form
the oxyanions:

, Re , ,
VI 2 VI I VI 2 VI I
4 4 4 4
Mo O Tc O W O O which are stable and in which the
transition elements concerned show their maximum oxidation states. The corresponding
oxyanions of Cr and Mn namely
VI 2
4
Cr O and
VII
4
Mn O are strong oxidizing agents.
Furthermore, the highet oxidation states of second and third row elements are encountered in
compounds containing the more electronegative elements viz. F,O and Cl. Thus for the
compounds , ,
VI I I VI I I
4 4
Ru O Os O
VI
6
W Cl and
VI
6
Pt F there are no analogs being formed by the
first row elements. The lower oxidation states particularly +2 and +3 are important in the
chemistry of aquated and complex ions of the 3d-series (i.e. first row) elements but these ions are
not very important in the chemistry of second (i.e. 4d series) and third (5d series) row elements.
In short it may be said that in going down asub-group the stability of the higher oxidation states
increases while that of lower oxidation states decreases.
iv) Formation of ionic and covalent compounds. Transition elements cannot form ionic compounds
in higher oxidation states because the loss of more than three electrons is prevented by the higher
attractive force exerted (on the electrons) by the nucleus. Higher oxidation states of these metals
are not formed by the actual loss of electrons but due to the formation of new hybrid orbitals
involving (n1)d, ns and np orbitals.
The transition elements form ionic bonds in the lower oxidation states and the ionic character of
the bond decreases with the increase in the oxidation state. With this decrease in the ionic
character of the bond the basic character of the oxides decreases, e.g. MnO (oxidation state of Mn
=+2) is basic,
2
MnO (Mn 4) = + is amphoteric and
3
MnO (Mn =+6) is acidic.
4.8 COLOUR
Compounds of transition elements are usually markedly coloured, in contrast to compounds of s- and
p-block elements which are mostly white or colourless unless the anion is coloured. As you know,
substances appear coloured when they absorb light of a particular wavelength in the visible region of
the spectrum and transmit light of other wavelengths. The colour which we see is the colour of the
transmitted wavelengths. In other words, the colour of the compound observed by us is the
complmentary colour of the colour absorbed by the compound.. You know that the transition metals as
such or in the form of ions have partly filled d-orbitals which are degenerate, i.e., they are of equal
nergy. In transition metal complexes the d-orbitals do not remain degenerate, but these split into sets of
orbitals of different energies. By absorbing energy, electrons can move from a d-orbital of lower
energy to that of higher energy. This transition of electron from one d-orbital to another corresponds to
a fairly small energy difference, therefore, light is absorbed in the visible region of spectrum. For
example, the aqua ion ( )
3
2 6
Ti H O
+
, which has one electron in the 3d orbital absorbs light of
wavelength in the yellow-green region of spectrum and therefore, appears reddish violet in colour.
Whenever the d-orbitals are completely filled or empty, there is no possibility of electronic transitions
within the d-orbitals. In such cases, the ions will not show any colour. For example, the compounds of
, ,
3 4
Sc Ti Cu
+ + +
and
2
Zn
+
are white or colourless.
In the s- and p-block elements there cannot be any d-d transitions and the energy needed to promate s
or p electron to a higher level is much greater and may correspond to ultraviolet region, in which case
the compound will not appear coloured to the eye.
Transition Element_6
Address: MHV Plaza, Krishi FarmRoad, Kela Nagar Aligarh, Phone: 0571-6543330/1
Relationship between colour and wavelength
Question 2: ZnO is yellow when hot, but white when cold.
Solution:
2
Zn
+
has
10
3d E.C. hence all electrons paired thus form white salt. The Yellow colour
of hot ZnO is due to defects in the solid structure which increase with temperature.
Exercise - 1
+
Cu is a
10
d ion and colourless but
2
Cu O is red and
2
Cu S is black.
4.9 MAGNETIC PROPERTIES
When a substance is placed in a magnetic field of strength H, the intensity of the magnetic field in the
substance may be greater than or less than H.
I f intensity >H , then substance is paramagnetic (Electrons unpaired).
I f intensity <H , then substance is diamagnetic (Electrons paired).
Substances which are weakly repelled by a magnetic field are called diamagnetic, white the
substances which are weakly attracted by the magnetic field and lose their magnetism when removed
from the field are called paramagnetic. If the force of attraction is very large and the permanent
magnetization is retained, the substance is said to be ferromagnetic, e.g., iron and some iron
compounds.
It should be noted that Fe,Co and Ni are ferromagnetic. Ferromagnetic materials may be regarded as
special case of paramagnetism in which the moments of individual atoms become aligned and all point
in the same direction. It is also possible to get antiferromagnetism by pairing the moments of adjacent
atoms which point in opposite directions. It occurs in salts of Fe
3+
, Mn
2+
etc.
Paramagnetism is expressed in terms of magnetic moment, which is related to the number of unpaired
electrons as follows
( ) n n 2 = + B.M.
n =number of unpaired electrons
B.M. =Bohr Magneton, unit of magnetic moment
More the magnetic moment more is the paramagnetic behaviour.
Question 3:
3
3 6
[Co(NH ) ]
+
isdiamagnetic and
3
6
[CoF ] is strongly paramagnetic.
Wavelength absorbed in nm Colour absorbed Colour observed
<400 UV region White/colourless
400435 Violet Yellow-green
435480 Indigo Yellow
480490 Green-blue Orange
490500 Blue-green Red
500560 Green Purple
560580 Yellow-green Violet
580595 Yellow Indigo
595605 Orange Green-blue
605750 Red Blue-green
>750 Infra-red White/colourless
Transition Element_7
Address: MHV Plaza, Krishi FarmRoad, Kela Nagar Aligarh, Phone: 0571-6543330/1
Solution: 3
Co
+
3
3 6
[Co(NH ) ]
+
3
6
[CoF ]
2 3
d sp
3 2
sp d
diamagnetic due to paired
electrons
Paramagnetic due to four unpaired electrons
3
CO
+
has
6
3d configuration with four unpaired electrons in ground state. In presence of
3
NH
(strong ligand) all the unpaired electrons in
3
Co
+
get paired and thus
3
3 6
[Co(NH ) ]
+
has
2 3
d sp
hybridization (octahedral structure), thus it is diamagnetic (no electron unpaired).

F is a weak
ligand hence six lone pairs of six

F are filled in outer d-orbitals of


3
Co
+
which has now four
electrons unpaired. Thus
3
6
CoF has
3 2
spd hybridization in
3
Co
+
and is thus paramagnetic due
to unpaired electrons.
Exercise - 2 Classify each of the following complexes as either high or low spin. Explain
you answers.

[ ( ) ] , . ;[ ( ) ] , .
2 4
2 6 6
Co H O 46BM Co CN 19BM
+
= =

[ ( ) ] , . ;[ ( ) ] , .
4 3
2 6 2 6
Fe NO 00BM Fe H O 594BM
+
= =
4.10 COMPLEX FORAMTION
The chemistry of the transition metals is dominated by their tendency to form complex ions. This is
because the transition elements form small, highly charged ions which have vacant orbitals of suitable
energy to accept lone pairs of electrons donated by other groups or ligands. In case of transition metals
in high oxidation states, highly charged inos can strongly bind electrostatically a wide variety of
negative or polar ligands. In the case of transition metals in low oxidation states, the electrons in the d
orbitals become involved in tbonding with ligands. The majority of transition metal ion complexes
contain six ligands surrounding the central ion octahedrally. Some elements contain four ligands which
are either arranged tetrahedrally or less frequently at the corners of a square. Besides these geometries,
other geometries like trigonal bipyramid, pentagonal bipyramid, etc., are also present occasionally. The
bonding between the ligand and the transition metal ion can either be predominantly electrostatic or
covalent or in many cases intermediate between the two extremes. Some of the typical complexes of the
transition meals are

[ ( ) ] ,[ ( ) ] ,[ ( ) ]
3 2 3
6 3 4 4
Fe CN Ni NH Cu CN
+
, [ ( ) ]
2
3 4
Cu NH
+
etc.
4.11 CATALYTIC PROPERTIES
Many transition metals and their compounds have catalytic properties. These metals can function as
catalysts because they can utilize both d and s electrons for the formation of bonds between reactant
molecules and the surface catalyst atoms. This increases the concentration of the reactants at the catalyst
surface and weakens the bonds in the reactant molecules with the result that the activation energy is
lowered. Compounds of transition metals are able to act as catalysts because of the case with which the
metal can adopt different oxidation states and also because of their ability to form complexes. Some of
the common catalysts used for important reactions.
3
TiCl Used as the Ziegler-Nattacatalyst in the production of polythene.
Transition Element_8
Address: MHV Plaza, Krishi FarmRoad, Kela Nagar Aligarh, Phone: 0571-6543330/1
2 5
V O
Converts
2
SO to
3
SO in the contact processfor making
2 4
H SO .
2
MnO
Used as a catalyst to decompose
3
KClO to give
2
O .
Fe
Promoted iron is used in the Haber-Bosch processfor making
3
NH .
3
FeCl
Used in the production of
4
CCl from
2
CS and
2
Cl .
4
FeSO
and
2 2
H O
Used as Fentons reagent for oxidizing alcohols to aldehydes.
2
PdCl
Wacker process for converting
2 4 2 2
C H H O PdCl + + to
3
CH CHO 2HCl Pd + +
Pd
Used for hydrogenation (e.g. phenol to cyclohexanone).
Pt/PtO
Adams catalyst, used for reductions.
Pt
Formerly used for
2 3
SO SO in the contact processfor making
2 4
H SO .
Pt
Is increasingly being used in three stage-convertors for cleaning car exhaust fumes.
Pt/Rh
Formerly used in the Ostwald processfor making
3
HNO to oxidize
3
NH to NO.
Cu
Is used in the direct process for manufacture of
3 2 2
(CH ) SiCl used to make silicones.
Cu/V
Oxidation of cyclohexanol/cyclohexanone mixtures to adipic acid which is used to make
nylon-66.
2
CuCl
Deacon processof making
2
Cl from HCl.
Ni
Raney nickel, numerous reduction processes (e.g. manufacture of hexamethylenediamine,
production of
2
H from
3
NH , reducing anthraquinone to anthraquinol in the production of
2 2
H O ).
Ni
complexes
Reppe synthesis(polymerization of alkynes) e.g. to give benzene or cyclooctatetraene.
4.12 INTERSTITIAL COMPOUND
Transition metals can trap some small atoms like hydrogen, boron, carbon, nitrogen, etc., in vacant
spaces in their crystal lattice forming interstitial compounds. Carbon and nitrogen always occupy
octahedral holes; hydrogen is smaller and always occupies tetrahedral holes. As only transition metals
form such compounds, the d electrons are, therefore, presumably involved in the bonding. The
structure of the meal often changes during the formation of such compounds. The composition of
these compounds is generally non-stoichiometric, e.g.,
. .
,
173 056
TiH PdH , but may approach regular
stoichiometry and a regular structure, e.g., TiC and VN. The later transition elements of the first series
form non-stoichiometric carbides with irregular structures, such as
7 3
Cr C , which are more reactive
than the interstitial carbides of the early transition elements. These interstitial compounds are of much
importance, e.g., carbon steels are interstitial iron-carbon compounds in which the interstitial carbon
prevents the iron atoms fromsliding over one another, making iron harder, stronger but more brittle.
The presence of these atoms results in decrease in malleability and ductility of the metals but increases
their tensile strength.
Transition Element_9
Address: MHV Plaza, Krishi FarmRoad, Kela Nagar Aligarh, Phone: 0571-6543330/1
5. INNER TRANSITION ELEMENTS
The elements in which the additional electrons enters (n2)f orbitals are called inner transition
elements. The valence shell electronic configuration of these elements can be represented as
(n2)
, ... , , ,
( )
02 14 012 2
f n 1d ns . These are also called f-block elements because the extra electron goes
to f-orbitals which belongs to (n2)th main shell. 4f-block elements are also called lanthanides or rare
earths. Similarly 5f-block elements are called actinides or actinones. The name Lanthanides and
actinides have been given due to close resemblance with Lanthanum and actinium respectively.
Lanthanides constitutes the first inner transition series while actinides constitutes second inner
transition series.
General Characteristics:
1. Electronic Configuration
n 1 0 2
[Xe] 4f 5d 6s
+
or
n 1 2
[Xe] 4f 5d 6s
2. Oxidationstate
They readily form
3
M
+
ions some of them also exhibit oxidation state of +2 and +4.
3. Colouration
Ions of Lanthanides and actinides are coloured in the solid state as well as in aqueous solution
because of absorbation of light due to ff transition since they have partly filled f-orbitals.
5.1 LANTHANIDE CONTRACTION
In lanthanides, the additional electron enters 4f-sub shell but not in the valence-shell namely sixth
shell. The shielding effect of one electron in 4f-sub-shell by another in the same sub-shell (i.e. mutual
shielding effect of 4f-electrons) is very little, being even smaller than that of d-electrons, because the
shape of f-sub shell is very much diffused. The nuclear charge (i.e. atomic number), however,
increases by unity at each step. Thus the nuclear charge increases at each step, while there is no
comparable increase in the mutual shielding effect of 4f-electrons. This results in that electrons in the
outermost shell experience increasing nuclear attraction from the growing nucleus. Consequently the
atomic and ionic radii go on decreasing as we move from
57
La to
71
Lu .
5.2 CONSEQUENCE OF LANTHANIDE CONTRACTION
1. Atomic and ionic radii of post-Lanthanide elements : The atomic radii of second row transition
elements are almost similar to those of the third row transition element because the increase in
size on moving down the group from second to third transition elements is cancelled by the
decrease in size due to lanthanide contraction.
2. High density of post lanthanide elements : It is because of very small size due to lanthanide
contraction.
3. Basic strength of oxides and hydroxides : Due to lanthanide contraction the decrease in size of
lanthanides ions, from
3
La
+
to
3
Lu
+
increases the covalent character (i.e. decreases the ionic
character) between
3
Ln
+
and

OH ions in Ln(III) hydroxides (Fajans rules). Thus


3
La(OH) is
the most basic while
3
Lu(OH) is the best basic.
Similarly, there is a decrease in the basic strength of the oxides.
4. Seperation of Lanthanides : Due to the similar size (Because of lanthanide contraction) of the
lanthanides, it is difficult to separate them. But slight variation in their properties is utilized to
separate.
Transition
Element_10
Address: MHV Plaza, Krishi FarmRoad, Kela Nagar Aligarh, Phone: 0571-6543330/1
6. COORDINATION COMPOUNDS
6.1 INTRODUCTION
The complexes show a wide variety of physical and chemical properties which are quite different from
normal salts. These difference arise due to the difference in their structures.
Molecular or addition compounds : When solution containing two or more salts in stoichiometric
(i.e., simple molecular) proportions are allowed to evaporate, we get crystals of compounds known as
molecular or addition compounds.
2 2 2 2
(carnallite)
KCl MgCl 6H O KCl.MgCl .6H O + +
2 4 2 4 3 2 2 4 2 4 3 2
(potassiumalum)
K SO Al (SO ) 24H O K SO .Al (SO ) .24H O + +
4 3 2 4 3 2
(tetramminecopper(II )sulphatemonohydrate)
CuSO 4NH H O CuSO .4NH .H O + +
2 2
(potassiumferrocyanide)
Fe(CN) 4KCN Fe(CN) .4KCN +
These are of two types depending on their behaviour in aqueous solution.
1. Double salts or Lattice compounds : The addition compounds having the following
characteristic are called double salts or lattice compounds.
a) They exist as such in crystalline state.
b) When dissolved in water, these dissociate into ions in the same way in which the individual
components of the double salts do.
2 2
4 4 2 4 2 4 4
Mohr'ssalt
FeSO .(NH ) SO .6H O Fe (aq) 2NH (aq) 2SO (aq)
+ +
+ +
3 2
2 4 2 4 3 2 4 2
Potashalum
K SO .Al (SO ) .24H O 2K (aq) 2Al (aq) 4SO (aq) 24H O
+ +
+ + +
2. Coordination (or complex) compounds. It has been observed that when solutions of Fe(
2
CN)
and KCN are mixed together and evaporated, potassium ferrocyanide,
2
Fe(CN) 4KCN is
obtained which in aqueous solution does not give test for the
2
Fe
+
and

CN ions, but gives the


test for K
+
ion and ferrocyanide ion,
4
2
Fe(CN) .
4
2 2 6
Fe(CN) 4KCN Fe(CN) .4KCN 4K Fe(CN)
+
+ +

Thus we see that in the molecular compound like


2
Fe(CN) 4KCN, the individual compounds lose
their identity. Such molecular compounds are called coordination (or complex) compounds. The
difference between a double salt and a complex compound appears to be one of the degree rather
than of a more fundamental unit.
A complex compound contains a simple cation and a complex anion or a complex cation and a
simple anion or a complex cation and complex anion, e.g.
IV IV
2 6 3 4 2 2
K [Pt Cl ],[PT (NH ) Br ]Br
and
III III
3 6 2 4 3
[CO (NH ) ][Cr (C O ) ] are all complex compounds. The term complex compound is
used synonymously with the term coordination compound.
In the above complex compounds the ions,

[ ] ,[ ( ) ] ,[ ( ) ]
I V 2 I V 2 I I I 3
4 3 4 2 3 6
Pt Cl Pt NH Br Co NH
+ +
and

[ ( ) ]
I I I 3
2 4 3
Cr C O are called complex ions. Thus a complex ion is an electrically charged
radical which is formed by the union of a metal cation with one or more neutral molecules or
anions.
Transition
Element_11
Address: MHV Plaza, Krishi FarmRoad, Kela Nagar Aligarh, Phone: 0571-6543330/1
7. IUPAC NOMENCLATURE OF COORDINATION COMPOUNDS
1. Naming of salt: If the complex is a salt, the cation is named first followed by the name of the
anion. For example, in naming
2 6
K [PtCl ] we shall name potassium first, followed by the name
of the anion. In another example of
3 4 2
[Cu(NH ) ]Cl , the name of the cation,
2
3 4
[Cu(NH ) ]
+
,
will be placed before chloride ion.
2. For the complex entity, whether it is in cationic, anionic or neutral form, the name of the
ligand(s) is put before the name of the metal atom. However, the reverse order is followed in
writing the formula of the compound. For example, in
3 4 2
[Cu(NH ) ]Cl , we shall name the four
ammonia molecules first followed by copper and finally the presence of chloride is mentioned;
but in writing the formula copper is written before ammonia molecules.
3. Naming of the negative ligands: The names of all anionic ligands end in `o replacing the final
`e in endings. Sometimes endings are also changed. Thus the given ligands acquire the following
endings:
Ligands Endings

F (fluoride) fluoro

Cl (chloride) choro
2
O (oxide) oxo

OCN (cyanate) cyanato


Cationic and neutral ligands have no special ending. There are a few exceptions. like `aqua for
2
H O, ammine for
3
NH , carbonyl for CO, and nitrosyl for NO groups.
4. I ndication of the number of ligands: The number of ligands is indicated by adding prefixes di,
tri, tetra, penta, hexa, etc. for two, three, four, five, six, etc entities of the ligand. For
example,
3 6 3
[Co(NH ) ]Cl will be called hexaamminecobalt chloride.
If the ligands are big complicated groups, instead of di, tri, tetra, penta prefixes we use bis,
tris, tetrakis, pentalkis etc. For instance
3 3 2
Cu(CH COCHCOCH ) is called
bis(acetylacetonato) copper.
5. Order of Naming ligands: In any complex species, the ligands are quoted in alphabetical order,
without regard to charge, before the name of the central metal atom. Numerical prefixes indicating
the number of ligands are not considered in determining that order. For example, a compound like
2 2
[CoCl(NO )(en) ]Cl will be called chlorobis (ethylenediamine) nitrocobalt(III) chloride.
6. Oxidation state: The oxidation state of the metal ion in a complex is indicated by Roman (I), (II),
(III) etc. or an Arabic (O) and placed in parenthesis immediately after thename of the metal. If,
however, the complex species is an anion, the oxidation state of the metal is mentioned at the end
of the name of the complex.
7. Naming of complex: The name of the complex anion always ends in ate and the Latin name of
the metal atom is used. No specific ending is used for neutral or cationic complex species. For
example,
2 6
K [PtCl ] is called potassium hexachloroplatinate(IV) and
2
K[Ag(CN) ] is named as
potassium dicyanoargentate(I).
8. A little space is given between the name of the cation and the anion. No space or hyphen is used
anywhere else.
9. Once the complex entity is completely identified according to the above rules, no mention of the
number of cations or anions used for charge balancing is required. For example,
3 6 3
[Co(NH ) ]Cl
is called hexaamminecobalt(III) chloride and not hexaamminecobalt(III) trichloride. Similarly,
2 6
K [PtCl ] is named potassium hexachloroplatinate(IV) and not dipotassium
hexachloroplatinate(IV).
Transition
Element_12
Address: MHV Plaza, Krishi FarmRoad, Kela Nagar Aligarh, Phone: 0571-6543330/1
10. Ligands having more than one donor atom: If a ligand has more than one donor atom, the
actual atom involved in the bond formation with the metal ion is indicated by putting italicized
symbol of the atom after the name of the ligand. For example,

[ ( ) ]
3
2 3 2
Ag S O is called
dithiosulphato s-argentate(I) ion. Some exceptions in this connection should be remembered.
These are:
SCN thiocyanato
NCS isothiocyanato

2
NO nitro
ONO nitrite
11. Naming of Bridging ligands: A bridging group is indicated by putting the Greek letter `
immediately before its name and separated by hyphens from other ligands. For example,
4
2 4 2 4
[(H O) Fe Fe(H O) ]
+
< >
OH
OH
is called `octaaqua- dihydroxodiiron(III) ion and the
formula could be written as
4
2 4 2 2 4
[(H O) Fe( OH) Fe(H O) ]
+
also.
12. Structural information may be given names and formulas by prefixes such as cis, trans etc.
3 2 2
[Pt(NH ) Cl ] can be written as cis-dichlorodiammineplatinum(II) or trans-
dichlorodiammineplatinum(II), respectively.
Note : When writing the formula of a complex, the central atom is listed first. The coordinated
groups (i.e., ligands) are listed in the order: formally anionic ligands, neutral ligands
followed by cationic ligands. Within each group, the ligands are listed alphabetically
according to the first symbol.
Question 4: Write down I PUAC name of K
2
[Fe(CN)
3
Cl
2
(NH
3
)
2
]
Solution: The positive part is named first followed by the negative part. In the negative part the names
are written in alphabetical order followed by metal. So the name is
Potassiumdiammlnedichlorotricyano-N-ferrate (III).
Question 5: Write the I UPAC name of [Co(NH
3
)
4
(NO
2
)
2
]Cl.
Solution: Here in this case the positive part is the complex. So it is name first with ligands in
alphabetical order followed by metal (but not ending in ate as the metal belong to the
positive part of the complex. This is followed by the negative part. So the name in
Tetraamminedinitrocobalt (III) chloride.
Exercise - 3 Write IPUAC name of [Pt(Py)
4
][PtCl
4
].
7.1 WERNERS THEORY OF COORDINATION COMPOUNDS
It was only in 1893, that Werner presented a theory known as Werner coordination theory which
could explain all the observed properties of complex compounds. More important postulates of this
theory are:
i) Most elements exhibt two types of valenceis : (a) primary valency and (b) secondary valency.
a) Primary valency. This corresponds to oxidation state of the metal ion. This is also called
principal, ionisable or ionic valency. It is satisfied by negative ions and its attachment with the
central metal ion is shown by dotted lines.
b) Secondary or auxiliary valency. It is also termed as coordination number (usually
abbreviated as CN) of the central metal ion. It is non-ionic or non-ionisable (i.e. coordinate
covalent bond type). This is satisfied by either negative ions or neutral molecules. The
ligands which satisfy the coordination number are directly attached to the metal atom or ion
and shown by thick lines. While writing down the formulae these are placed in the
coordination sphere along with the metal ion. These are directed towards fixed position in
space about the central metal ion, e.g. six ligands are arranged at the six corners of aregular
octahedron with the metal ion at its centre. This postulate predicted the existence of
different types of isomerism in coordination complexes and after 19 years Werner actually
succeeded in resolving various coordination examples into optically active isomers.
Transition
Element_13
Address: MHV Plaza, Krishi FarmRoad, Kela Nagar Aligarh, Phone: 0571-6543330/1
.
3 3
CoCl 6NH or
Co
H
3
N
H
3
N
Cl
NH
3
NH
3
Cl
Cl
NH
3
NH
3

[( ( ) ] ( )
I I I 3
3 6 3
Co NH Cl
+
Co
H
3
N
H
2
O
Cl
NH
3
NH
3
Cl
Cl
NH
3
NH
3
.
3 3 2
CoCl 5NH H Oor

[( ( ) ( )] ( )
I I I 3
3 5 2 3
Co NH H O Cl
+
.
3 3
CoCl 5NH or
Co
H
3
N
H
3
N
Cl
NH
3
Cl
NH
3
NH
3
Cl

[( ( ) ] ( )
I I I 2
3 5 2
Co NH Cl Cl
+
.
3 3 2
CoCl 4NH H Oor

[( ( ) ]
I I I
3 4 2
Co NH Cl Cl
+
Co
H
3
N
H
3
N
Cl
Cl
NH
3
Cl
NH
3
. [ ( ) ]
I II 0
3 3 3 3 3
CoCl 3NH or Co NH Cl
Co
H
3
N
Cl
Cl
NH
3
Cl
NH
3
ii) Every element tends to satisfy both its primary and secondary vlencies. In order to meet this
requirement a negative ion may often show a dual behaviour, i.e. it may satisfy both primary and
secondary valencies since in every case the fulfillment of coordination number of the central
metal ion appears essential.
Characteristic of Co(I I I) ammines
7.2 EFFECTIVE ATOMIC NUMBERS
The number of secondary valencies in the Werners theory is now called the coordination number of
the central metal in the complex. This is the number of ligand atoms bonded to the central metal ion.
Each ligand donates an electron pair to the metal ion, thus forming a coordinate bond. Transition
Ammines (i.e.
complexes
No. of

Cl
ions
precipitated
as AgCl by
3
AgNO
solution
Molar
conducti
vity
range
1
(ohm )
Total No.
of ions
given by
complex in
soln.
Charge
type on
ions
I onic Formulation
3 3
CoCl .6NH
3 430 4 (3+, 1)
III 3
3 6 3
[Co (NH ) ] (Cl )
+
3 3 2
CoCl .5NH .H O
3 430 4 (3+, 1)
III 3
3 5 2 2
[Co (NH ) (H O)] (Cl )
+
3 3
CoCl .5NH
2 250 3 (2+, 1)
III 2
3 5 2
[Co (NH ) Cl] (Cl )
+
+
3 3
CoCl .4NH
1 100 2 (1+, 1)
III
3 4 2
[Co (NH ) Cl ] Cl +
3 3
CoCl .3NH
0 0
III 10
3 3 3
[Co (NH ) Cl ]
(Non.
electrolyte)
(Non-electrolyte)
Transition
Element_14
Address: MHV Plaza, Krishi FarmRoad, Kela Nagar Aligarh, Phone: 0571-6543330/1
metals form coordination compounds very readily because they have vacant d orbitals which can
accommodate these electron pairs. The electronic arrangement of the noble gases is known to be very
stable. Sidgwick, with his effective atomic number rule, suggested that electron pairs from ligands
were added until the central metal was surrounded by the same number of electrons as the next noble
gas. Consider potassium hexacyanoferrate(II) [ ( ) ]
4 6
K FeCN (formerly called potassium
ferrocyanide). An iron atom has 26 electrons, and so the central metal ion
2
Fe
+
has 24 electrons. The
next noble gas Kr has 26 electrons. Thus the addition of six electron pairs from six

CN ligands adds
12 electtrons, thus raising the effective atomic number (EAN) of
2
Fe
+
in the complex
4
6
[Fe(CN) ] to
36.
[24 (6 2) 36] + =
The EAN rule correctly predicts the number of ligands in many complexes. There are, however, a
significant number of exceptions where the EAN is not quite that of a noble gas. If the original metal
ion has an odd number of electrons, for example, the adding of electron pairs cannot result in a noble
gas structure. The tendency to attain a noble gas configuration is a significant factor but not a
necessary condition for complex formation. It is also necessary to produce a symmetrical structure
(tetrahedral, square planar, octahedral) ireespective of the number of electrons involved.
Effective atomic numbers of some metals in complexs
Atom
Atomic
number
Complex
Electrons lost
in ion
formation
Electrons gained
by coordination
EAN
Cr 24
6
[Cr(CO) ]
0 12 36
[Kr]
Fe 26 4
6
[Fe(CN) ]
2 12 36
Fe 26
5
[Fe(CO) ]
0 10 36
Co 27 3
3 6
[Co(NH ) ]
+ 3 12 36
Ni 28
4
[Ni(CO) ]
0 8 36
Cu 29 3
4
[Cu(CN) ]
1 8 36
Pd 46 4
3 6
[Pd(NH ) ]
+ 4 12 54 [Xe]
Pt 78 2
6
[Pt(Cl ]
4 12 86 [Rn]
Fe 26 3
6
Fe(CN) ]
3 12 35
Ni 28 2
3 6
[Ni(NH ) ]
+ 2 12 38
Pd 46 2
4
[PdCl ]
2 8 52
Pt 78 2
3 4
[Pt(NH ) ]
+ 2 8 84
7.3 SHAPES OF d ORBITALS
Since d orbitals are often used in coordination complexes it is important to study their shapes and
distribution in space. The five d orbitals are not identical and theorbitals may be divided into two sets.
The three
2g
t orbitals have identical shape and point between the axes, x, y and z. The two
g
e orbitals
have different shapes and point along the axes. Alternative names for
2g
t and
g
e are dc and d
respectively.
Transition
Element_15
Address: MHV Plaza, Krishi FarmRoad, Kela Nagar Aligarh, Phone: 0571-6543330/1
x
2g
t
orbitals
(d ) c
x y
y
z z
xy
d
xz
d
2
z
d
yz
d
2 2
x y
d
x
y
z
g
e
orbitals
(d )
8. ISOMERISM
Compounds that have the same chemical formula but different structural arrangement are called
isomers. Because of the complicated formula of many coordination compounds,the variety of bond
types and the no of shapes possible, many different types of isomerism occur.
[A] STRUCTURAL I SOMERSI M
(I ) Polymerization I somerism: This is not true isomerism because it occurs between compounds
having the same empirical formula, but different molecular weights. For example,
[Pt(NH
3
)
2
Cl
2
], [Pt(NH
3
)
4
][PtCl
4
], [Pt(NH
3
)
4
][Pt(NH
3
)Cl
3
]
2
.
(I I ) I onization I somerism : This type of isomerism is due to the exchange of groups between the
complex ion and the ions outside it. [Co(NH
3
)
5
Br]SO
4
is red violet. An aqueous solution gives
a white precipitate of BaSO
4
with BaCl
2
solution, thus confirming the presence of free
2
4
SO ions. In contrast [Co(NH
3
)
5
SO
4
]Br is red. A solution of this complex does not give a
positive sulphate test with BaCl
2
. It does give a cream coloured precipitate of AgBr with
AgNO
3
, thus confirming the presence of free Br

ions.
(I I I ) Hydrate I somerism : Three isomers of CrCl
3
.6H
2
O are known. From conductivity
measurements and quantitative precipitation of the ionized chlorine, they have been given the
following formulae:
[Cr(H
2
O)
6
]Cl
3
violet (three ionic chlorines)
[Cr(H
2
O)
5
Cl]Cl
2
.H
2
O green (two ionic chlorines)
[Cr(H
2
O)
4
Cl
2
].Cl.2H
2
O dark green (one ionic chlorine)
(I V) Linkage I somerism : Certain ligands contain more than one atom which could donate an
electron pair. In the

2
NO ion, either N or O atoms could act as the electron pair donor. Thus
there is the possibility of isomerism. Two different complexes [Co(NH
3
)
5
NO
2
]Cl
2
have been
prepared, each containing the

2
NO group in the complex ion.
(V) Coordination I somerism : If the complex is a salt having both cation and anion as complex
ions then the ligands can exchange position between the cation and the anion. This will result in
the formation of coordination isomers. For example
3 2 4 3 2 2 4 2 4 2
[Co(en) ][Cr(C O ) ]and[Co(en) (C O )][Cr(en)(C O ) ]
2 2 4 2 4 2 3 2 4 3
[Cr(en) (C O )][Co(en)(C O ) and[Cr(en) ][Co(C O ) ]
(VI ) Coordination Position I somerism : If in a multinuclear complex the distribution of ligands
around the metal centres changes it will result in a different isomer. Such an isomerism is called
coordination position isomerism. Some typical examples are :
Transition
Element_16
Address: MHV Plaza, Krishi FarmRoad, Kela Nagar Aligarh, Phone: 0571-6543330/1
3 4 3 2 2 2 3 3 3 3 2
[(NH ) Co Co(NH ) Cl ]Cl and [Cl(NH ) Co Co(NH ) Cl]Cl < > < >
2
2
NH
O
2
2
NH
O
3 2 2 3 3
[(R P) Pt PtCl ] and [Cl(R P) Pt Pt(R P)Cl] < > < >
Cl
Cl
Cl
Cl
(VI I ) Electronic I somerism: The complex
3 5 2
[Co(NH ) NO]Cl exists in two forms. One is black
paramagnetic while the other is pink and diamagnetic. Thorough structural studies show that the
black isomer is a Co(II) complex containing neutral NO group whereas the pink one is a Co(III)
complex where

NO ion is present. This kind of isomerism is known as electronic isomerism.


[B] STEREO I SOMERISMS
Stereoisomers have the same bonds but the arrangement of atoms in space is different.
Stereoisomerism can be divided into two kinds: geometrical and optical.
(I ) Geometrical isomerism or cis-trans isomerisms: It occurs when ligands can assume different
positions around rigid bonds with the metal ion. For example, the compound
3 2 2
Pt(NH ) Cl has
a square planar structure. The two possible arrangements are.
Pt
3
H N
3
H N Cl
Cl
Pt
3
H N
3
NH
Cl
Cl
In the cis-isomer the two ammonia molecules are next (cis) or adjacent to each other whereas in
the trans-isomer the two ammonia molecules are across (trans) to each other. This type of
isomerism is not possible for complexes with coordination number 2 (linear molecule), 3
(trigonal planar) and 4 (tetrahedral geometry).
For square planar complexes
4 3
Ma ,Ma b or
3
Mab where a and b etc., are monodentate
ligands, again the geometrical isomerism is not possible. However, square planar complexes of
the type
2 2
Ma b ,
2
Ma bc, Mabcd and
2
M(AA) ,
2
M(AB) where AA and AB represent
symmetrical and unsymmetrical chelating agents do give geometrical isomers. A few
examples are given below:
Type Compound I somers
2 2
Ma b
3 2 2
Pt(NH ) Cl
Pt
3
H N
3
H N Cl
Cl
Pt
3
H N
3
NH
Cl
Cl
2
Ma bc
3 2
Pt(NH ) ClBr
Pt
3
H N
3
H N Br
Cl
Pt
3
H N
3
NH
Br
Cl
Transition
Element_17
Address: MHV Plaza, Krishi FarmRoad, Kela Nagar Aligarh, Phone: 0571-6543330/1
Mabcd
3 5 5
Pt(NH )(C H N)(Cl)(Br)
Pt
3
H N
Br Cl
Pt
3
H N 5 5
NC H
Cl
5 5
NC H
Br
Pt
3
H N
5 5
NC H
Cl
Br
Bridged binuclear planar complexes like [( ) ]
3 2 2
Pt PEt Cl may exist in three isomeric forms:
Cl
Pt
Cl
Pt
PEt
3
Et
3
P Cl Cl
Cl
Pt
Cl
Pt
Cl
Et
3
P Cl PEt
3
Et
3
P
Pt
Cl
Pt
Cl
Et
3
P Cl Cl
trans
cis unsymmetrical
Six coordinated octahedral complexs of the type , , , ,
4 2 3 3 3 2 3 2 2
Ma b Ma b Ma b c Ma bcd Ma b cd,
,
2
Ma bcde Mabcdef would all give geometrical isomers. Systems with one or two bidentate ligands
and rest monodentate would also give geometrical isomers. Thus we see that with this geometry a lage
number of isomers are possible whether they can be isolated or separated is a different question which
depends on so many factors. As we increase the number of different ligands, the possible number of
isomers increases. For example,
4 2
Ma b type of complex would give only two isomers cisand trans.
Similarly for
3 3
Ma b type of complex we again get two isomers facial (fac) and meridional (mer)
isomers. In the former (fac) three ligands of one type form one triangular face of the octahedron and
the other three on the opposite face. In the latter (mer) one set of these ligands are arranged around an
edge of the octahedron whereas the other set occupies the opposite edge as shown in figure.
M
a
a b
b
b
a
M
a b
b
a
b
a
Facial and meridional isomers of
3 3
Ma b complex
(I I ) Optical isomerism: Two isomers whichhave almost identical physical and chemical properties
like mp, bp, density, colour etc., but differ in the way they rotate the plane-polarised light are
called optical isomers. Such optically active compounds exist in pairs and are known as
stereoisomers or enantiomers. These isomers aer non-superimposable mirror images of each
other. Hence, any molecule which contains either a centre of symmetry or a plane of symmetry
will not show optical isomerism.
Optical isomerismis rarely observed in square planar complexes. Tetrahedral complexes of the
type [ ( ) ]
2
M AB do give optical isomers as shown in figure especially where M =Be, B etc.
M
A
B
A
B
M
A
B
A
B
Transition
Element_18
Address: MHV Plaza, Krishi FarmRoad, Kela Nagar Aligarh, Phone: 0571-6543330/1
However, optical isomerism is very common with octahedral complexes of the type
( ) , ( ) , ( ) ,
3 2 2 2
M AA M AA ab M AA a b ( )( )
2
M AA BB a etc. A few typical examples are shown in
figure.
Co
2
H N
2
H N
2
H N
2
H N
2
H C
2
H C
2
CH
2
CH
2
CH
2
CH
Co
N
2
NO
2
NO
N
N
N
en
en
Co
N
2
O N
2
O N
N
N
N
en
en
Co
2
H N
2
NH
2
H C
2
H C
2
CH
2
CH
2
CH
2
CH
3+ 3+
+ +
2
NH
2
NH
2
H N
2
H N
2
NH
2
NH
Question 6: How do you distinguish between the following pairs of isomers?
i) [ ( ) ] [ ( ) ]B
3 5 3 5
Cr NH Br Cl and Cr NH Cl r
ii) [ ( ) ][ ( ) ] [ ( ) ][ ( ) ]
3 6 2 6 3 6 2 6
Co NH Cr NO and Cr NH Co NO
Solution: i) The isomers can be distinguished by using
3
AgNO reagent. One gives curdy
precipitate of AgCl soluble in ammonia while the other will form light yellow
precipitate of AgBr partly soluble in ammonia.
ii) The isomers can be distinguished by passing their aq. solutions through a cation
exchanger. In the isomer
3 6 2 6
[Co(NH ) (Cr[NO ) ] the cation
3
3 6
[Co(NH ) ]
+
will be replaced by H
+
ions from the resin. The resulting solution will thus
contain
3 2 6
H [Cr(NO ) ]. In the other case, the resulting solution after passage
through the exchanger will contain
3 2 6
H [Co(NO ) ]. On adding KCl solution,
3
2 6
[Co(NO ) ] ion will give yellow precipitate of potassium cobaltinitrate,
3 2 6
K [Co(NO ) ]. Hence the resulting solutions obtained from the two isomers
can be distinguished.
9. HYBRIDIZATION AND GEOMETRY
9.1 FORMATION OF AN OCTAHEDRAL COMPLEX
Let us consider the case of six ligands forming an octahedral complex. For convenience, we may
regard the ligands as being symmetrically positioned along the axes of a Cartesian co-ordinate system
with the metal ion at the origin. To simplify the situation, we can consider an octahedral complex as a
cube, having the metal ion at the centre of the body and the 6 ligands at the face centres.
and if we take the metal ion as the origin of a Cartesian co-ordinate, the ligands will be along the axes.
As in the case of a spherical field, all of the d-orbitals will be raised in energy relative to the free ion
because of negative charge repulsions. However it should be pictorially obvious that not all of the
Transition
Element_19
Address: MHV Plaza, Krishi FarmRoad, Kela Nagar Aligarh, Phone: 0571-6543330/1
orbitals will be affected to the same extent. The orbitals lying along the axes ( )
2
y
2
x
2
z
d and d

will be
more strongly repelled than the orbitals with lobes directed between the axes (d
xy
, d
xz
, d
yz
). The d-
orbitals are thus split into two sets with the

2 2 2
z x y
d andd at a higher energy than the other three.
( )
2
y
2
x
2
z
d , d

d
xy
, d
yz
, d
xz
A
0
[A] Factors affecting the magnitude of
0
i) Oxidation state of the metal ion: The magnitude of A
0
increases with increasing ionic charge on
the central metal ion. As the ionic charge on the metal ion increases greater is the attraction for the
ligands, greater the repulsion and hence greater the magnitude of A
0
.
ii) Nature of the ligands: Based on experimental observation for a wide variety of complexes, it is
possible to list ligands in order of increasing field strength in a spectrochemical series. Although it
is not possible to form a complete series of all ligands with a single metal ion, it is possible to
construct one from overlapping sequences, each constituting a portion of the series:
I

Br

S
2
SCN

Cl

N
3

, F

urea, OH

ox, O
2
, H
2
O NCS

py, NH
3
en bpy, phen NO
2

C
6
H
5

CN

CO.
The spectrochemical series and other trends described allow one to rationalise differences in spectra
and permit some predictabiltiy. If the splitting of the d-orbitals resulted simply from the effect of
point charges (ions (or) dipoles), one should expect that anionic ligands would exert the greatest
effect. To the contrary most anionic ligands lie at the low end of the spectrochemical series. Further
more, OH

lies below the neutral H


2
O molecule and NH
3
produces a greater splitting than H
2
O.
Despite its imperfections, the basic theory can be used to interpret a number of effects in co-
ordination chemistry.
[B] Outer orbital and I nner orbital complexes
Consider the complexes [CoF
6
]
3
and [Co(NH
3
)
6
]
3+
The electronic configuration of Co
3+
ion is
3d 4s 4p 4d

In a weak ligand field such as [CoF
6
]
3
, the approach of the ligand causes only a small split in the
energy level.
Transition
Element_20
Address: MHV Plaza, Krishi FarmRoad, Kela Nagar Aligarh, Phone: 0571-6543330/1
Since the ligand is a weak field ligand, its repulsions with the
electrons in
2
y
2
x
2
z
d and d

orbitals are very less (or) in other words
we can say that the electrons in
2
z
d and
2
y
2
x
d

cannot move away
from the approaching ligands since they have insufficient energy to
pair up with the electrons in d
xy
, d
yz
and d
xz
orbitals.
Thus there are no vacant orbitals in the 3d shell and the ligands occupy the first six vacant orbitals (one
4s, three 4p and two 4d). Since outer d orbitals are used, this is an outer orbit complex. The orbitals are
hybridised and are written sp
3
d
2
to denote this. Since none of the electrons has been forced to pair off,
this is a high spin complex and will be strongly paramagnetic because it contains 4 unpaired 3d
electrons.
Under the influence of a strong ligand field as in the complex [Co(NH
3
)
6
]
3+
, the approach of the ligand
causes a greater split in the energy level.
Since, the split is very high, we can say that the energy difference
between the two sets of orbitals is much greater and this energy
difference is sufficient to allow the electrons in
2
y
2
x
2
z
d and d

orbitals to move into the half filled d
xz
, d
xy
and d
yz
orbitals, even
though this pairing requires energy. We can also view this like, the
ligand repel the electrons in higher energy level to an extent such
that they get paired up against Hund's rule
So,
The
2
z
d and
2
y
2
x
d

orbitals become vacant.
The six ligands each donate a lone pair to the
first six vacant orbitals, which are: two 3d, one
4s and three 4p. Inner d-orbitals are used and
so this is an inner orbital complex. The orbital
are hybridised and written d
2
sp
3
to denote the
use of inner orbitals.
Since, the orginal unpaired electrons have been forced to pair off, there is a low spin complex and is in fact
diamagnetic.
The inner and outer orbital complexes may be distinguished by magnetic measurements. Since the outer
orbital complexes use high energy levels, they tend to be more reactive. The inner orbitals are sometimes
called inert orbitals.
Transition
Element_21
Address: MHV Plaza, Krishi FarmRoad, Kela Nagar Aligarh, Phone: 0571-6543330/1
Distribution of d-electrons in
2g
t and
g
e sets in strong(er)
and weak (er) octahedral ligand fields.
I ons
Strong(er) field (low-spin or spin-paired
complexes)
0
( P) A >
Weak(er) field (high-spin or spin free
complexes)
0
( P) A <
p q
2g g
t e
Configuration
n s
p q
2g g
t e
configuration
n S
1
d
2
d
3
d
1 0
2g g
t e
2 0
2g g
t e
3 0
2g g
t e
1
2
3

1
3/2
1 0
2g g
t e
2 0
2g g
t e
3 0
2g g
t e
1
2
3

1
3/2
4
d
5
d
6
d
7
d
4 0
2g g
t e
5 0
2g g
t e
6 0
2g g
t e
6 1
2g g
t e
2
1
0
1
1

0
1/2
3 1
2g g
t e
3 2
2g g
t e
4 2
2g g
t e
5 2
2g g
t e
4
5
4
3
2
5/2
2
3/2
8
d
9
d
10
d
6 2
2g g
t e
6 3
2g g
t e
6 4
2g g
t e
2
1
0
1

0
6 2
2g g
t e
6 3
2g g
t e
6 4
2g g
t e
2
1
0
1

0
9.2 FORMATION OF A SQUARE PLANAR COMPLEX
If the central metal ion has eight d-electrons, these will be arranged as

In a weak octahedral ligand field, a regular octahedral complex is thus formed by using outer d-
orbitals.
However, under the influence of a strong ligand field, the electrons in the
2
z
d and
2 2
y x
d

orbitals may
pair up, leaving one vacant d-orbital, which can accept a lone pair from a ligand.
For example consider [Ni(CN)
4
]
2
The electronic configuration of Ni
2+
ion is
3d 4s 4p

Consider, a Ni
2+
ion with one electron in the
2 2
y x
d

orbital and one
in the
2
z
d orbital. The approach of ligands along x, y and z axes
will result in increase in the energy of these orbitals. Because the
2
y
2
x
d

orbital is attacked by four ligands and the
2
z
d by only two,
the energy of
2
y
2
x
d

orbital will increase most. If the ligands have
enough strong field, the electrons will be forced out of the
2
y
2
x
d

orbital into the
2
z
d .
Transition
Element_22
Address: MHV Plaza, Krishi FarmRoad, Kela Nagar Aligarh, Phone: 0571-6543330/1
Thus four ligands can approach along x, x, y and y axes. A ligand approaching in the z (or) z
direction attempting to donate a lone pair meets the very strong repulsive forces from a completely
filled
2
Z
d orbital. Thus only four ligands succeed in bonding to the metal.
9.3 FORMATION OF TETRAHEDRAL COMPLEXES
A regular tetrahedron is related to a cube with an atom at the centre and four of the eight corners
occupied by ligands.
y
x
z
The directions x,y and z point to the centre of the faces. The
2
Z
d and
2
y
2
x
d

orbitals point along x,y
and z axis and d
xy
, d
yz
and d
xz
orbitals point in between x,y andz.
The directions of approach of the ligands does not coincide exactly with either the
2
z
d and
2
y
2
x
d

orbitals (or) the d
xy
, d
yz
and d
xz
orbitals. The approach of ligands raises the energy of both sets of
orbitals, but since the d
xy
,d
yz
and d
xz
orbitals correspond more closely to the position of the ligands,
their energy increases most and the
2
Z
d and
2
y
2
x
d

orbitals are filled first. This is opposite to what
happens in octahedral complexes.
Consider, the complex ion, [FeCl
4
]

. The electronic configuration of Fe


3+
ion is
3d 4s 4p

Since Cl

ion is a weak field ligand it is unable to pair the unpaired electrons and hence, the Cl

ion
uses 4s and 4p orbitals to form atetrahedral complex of sp
3
hybridisation.
10. CHELATION
Polydentate ligands whose structures permit the attachment of their two or more donor atoms (or sites)
to the same metal ion simultaneously and thus produce one or more rings are called chelate or
chelating ligands (from the Greek for claw) or chelating groups. However, it should be noted that every
multidentate ligand is not necessarily a chelating ligand the coordinating atoms of the ligand may be
so arranged that they cannot be coordinated to thesame metal atom to produce a ring structure. Thus

2 2 2 2
NH CH CH NH is a chelating ligand, while
2 2 2
NCH CH NH
2 2
(CH )
2 2
(CH )
is not, although both are diamines. The formation of such rings is termed chelation and the resulting
ring structures have been called chelate rings or simply chelates by Morgan and Drew. These are also
called chelated complexes or cyclic complexes and the term chelation is also called cyclisation.
10.1 CLASSIFICATION OF CHELATES
Since the polydentate ligands may be attached to the central metal ion through two kinds of functional
groups namely acidic and coordinating groups to form covalent and coordinate linkages respectively,
the classification of chelates follows the number and kind of linkages by which the metal ion is
attached with the ligands. The covalent bonds are formed by the replacement of one or more H-atoms
from the acidic groups present in the ligand by the metal atom. The examples of most common grops
of this type are COOH (carboxyl),
3
SO H (sulphonic), OH (enolic hydroxyl) and =NOH
(oxime). Coorinating linkages, without the replacement of H, are formed by the donation of an electron
pair from the ligands. The examples of most common groups of this type are
2
NH (primary,
Transition
Element_23
Address: MHV Plaza, Krishi FarmRoad, Kela Nagar Aligarh, Phone: 0571-6543330/1
secondary and tertiary amines), =NOH (oxime), OH (alcoholic hydroxyl), =CO (carbonyl) and
S(thio ether).
Ordianrily such chelate rings as mentioned above are most stable, because of reducd strain, when they
have 5 or 6 members includingthe metal ion. The enhanced stability of complexes containing chelated
ligads (i.e., multidentate ligands) is known as the chelate effect.
11. SOLVED OBJECTIVE QUESTIONS
Question1: Which electronic configuration represents a transition element?
(a)
2 2 6 2 6 10 2 6
1s 2s 2p 3s 3p 3d 4s 4p
(b)
2 2 6 2 6 10 2 6
1s 2s 2p 3s 3p 3d 4s 4p
(c)
2 2 6 2 6 2 2
1s 2s 2p 3s 3p 3d 4s
(d)
2 2 6 2 6 2
1s 2s 2p 3s 3p 4s .
Solution: (c)
Question2: Transition metals are less reactive because of their
(a) High I .P. and low melting point
(b) High I .P. and high melting point
(c) Low I .P. and low melting point
(d) Low I .P. and high melting point.
Solution: (b)
Question3: The stable oxidation states of Mn are
(a) +2, +3 (b) +2, +7
(c) +3, +7 (d) +3, +5.
Solution: (b).
Question4: Which is not true statement about FeO?
(a) it is non-stoichiometric and is metal deficient
(b) it is basic oxide
(c) its aqueous solution changes to ( )
3
Fe OH and then to .( )
2 3 2 n
FeO H O by
atmospheric oxygen
(d) it gives red colour with KCNS.
Solution: (d)
Question5: Paramagnetism is given by the relation ( ) 2s s 1 = + magnetons where `s is the
total spin. On this basis, the paramagnetism of Cu
+
ion is:
(a) 3.88 magnetons (b) 2.83 magnetons
(c) 1.41 magnetons (d) zero.
Solution: (d)
Question6: Fe is made passive by:
(a) dil
2 4
H SO (b) dil HCl
(c) aqua regia (d) conc.
2 4
H SO
Solution: (c)
Question7: Maximum magnetic moment is shown by:
(a)
5
d (b)
6
d
(c)
7
d (d)
8
d .
Solution: (a)
Question8:

4
MnO is of intense pink colour, though Mn is in (+7) oxidation state. I t is due to
(a) oxygen gives colour to it
(b) charge transfer when Mn gives its electron to oxygen
(c) charge transfer when oxygen gives its electron to Mn making it Mn (+VI ) hence
coloured
(d) None is correct.
Transition
Element_24
Address: MHV Plaza, Krishi FarmRoad, Kela Nagar Aligarh, Phone: 0571-6543330/1
Solution: (c)
Question9: Out of

, , , , ,
3 2 2
4 4 4 4 4 4
SiCl TiCl PO SO CrO CCl isostructrual are:
(a) ,
4 4
SiCl TiCl (b)

,
2 2
4 4
SO CrO
(c) both (d) none of these.
Solution: (c)
Question10: I ncreasing basic properties of ,
2 2
TiO ZrO and
2
HfO are in order:
(a)
2 2 2
TiO ZrO HfO < < (b)
2 2 2
HfO ZrO TiO < <
(c)
2 2 2
HfO TiO ZrO < < (d)
2 2 2
ZrO TiO HfO < < .
Solution: (a)
Question11: The transition elements are so named because
(a) they have partly filled d-orbitals
(b) their properties are similar to other elements
(c) their properties are different from other elements
(d) they lie between the s- and p-blocks
Solution: (d)
Question12: CrO
3
dissolves in aqueous NaOH to give
(a) Cr
2
O
7
2
(b) CrO
4
2
(c) Cr(OH)
3
(d) Cr(OH)
2
Solution: (b)
Question13: Transition elements are coloured due to
(a) small size (b) metallic nature
(c) unpaired d-electrons (d) none of these
Solution: (c)
Question14: Choose the correct answer for transition elements
(a) they have low melting points
(b) they do not exhibit catalytic activity
(c) they exhibit variable oxidation states
(d) they exhibit inert pair effect.
Solution: (c)
Question15: Pt, Pd and I r are called noble metals because
(a) Alfred Nobel discovered them
(b) they are found in active states
(c) they are inert towards many common reagents
(d) they are shining, lustrous and pleasant to look at
Solution: (c)
12. SOLVED SUBJECTIVE QUESTIONS
Question 1: Calculate the magnetic moments of Fe
2+
and Fe
3+
Solution: In
2
Fe
+
there are 4 unpaired electrons.
4(4 2) 4 6 24 4.89B.M. = + = = =
In
3
Fe
+
there are 5 unpaired electrons.
5(5 2) 5 7 35 5.91B.M. = + = = =
Question2: When Zn is added to an acidified solution of
2 2 7
K Cr O , the colour of the solution
changes from orange to green, then to blue and over a period of time, back to green.
Write equations for this series of reactions.
Solution: Zn reduces
2
2 7
Cr O (orange)/ H
+
to
3
Cr
+
(green) and then to
2
Cr
+
(blue), but
atmospheric
2
O oxides
2
Cr
+
(blue) back to
3
Cr
+
(green):
Transition
Element_25
Address: MHV Plaza, Krishi FarmRoad, Kela Nagar Aligarh, Phone: 0571-6543330/1
2
Zn Zn 2e
+
+
2 3
2 7 2
Cr O 14H 6e 2Cr 7H O
+ +
+ + +
3 2
Cr e Cr
+ +
+
2 3
2 2
air
4Cr O 4H 4Cr 2H O
+ + +
+ + +
Question3: a) CO and

CN are toxic. Explain by a suitable example.


b)
2
2 8
S O (peroxy disulphate ion) oxidizes Ag
+
in the presence of excess
pyridine. Write chemical reaction.
Solution: a) CO is toxic because it forms a complex with iron of haemoglobin in the blood,
and this complex is more stable than oxy-haemoglobin. This prevents the
haemoglobin in the red blood corpuscles from carrying oxygen round the body.
This causes an oxygen deficiency, leading to unconsciousness and then death.
The extreme toxicity of cyanides is due to

CN complexing with metals in


cytochrome-enzymes and haemoglobin in the body, thus preventing normal
metabolism.
b)
2 2 2
2 8
2Ag S O 2Ag 2SO
+ +
+ +
2 2
2
red
Ag 4Py [Ag(Py) ]
+ +
+
Question4: Explain the following
i) Anhydrous FeCl
3
cannot be obtained by heating hydrated FeCl
3
.
ii) The colour of mercurous chloride changes from whiteto black when treated
with ammonia solution.
iii) The compounds of Zn, Cd and Hg are usually white.
iv) A drak blue precipitate is formed when NaOH solution is added to CuSO
4
solution. The precipitate darkens on heating.
Solution: i) On heating hydrated
3
FeCl , anhydrous
3
FeCl is not formed as water of
crystallization reacts to form
2 3
FeO and HCl.
ii)
2 2
Hg Cl reacts with
3
NH to form a mixture of mercury and mercuric
aminochloride which is a black substance.
2 2 3 2
black
Hg Cl NH Hg Hg(NH )Cl HCl + + +

iii) In the compounds of zinc metals,


2
M
+
ions possess the penultimate d-orbitals
doubly occupied i.e.,
10
(n1)d configuration. There is no d-d transition. Hence
the compounds of zinc metals are colourless.
iv) NaOH reacts with
4
CuSO when dark blue precipitate of
2
Cu(OH) is formed.
This precipitate of heating forms CuO which is black in colour. Hence, the colour
darkens on heating.
4 2 2 4
CuSO 2NaOH Cu(OH) Na SO + +
2 2
Black
Cu(OH) CuO H O +
Question5: Explain the following
i) Copper (l) salts are not known in aqueous solutions.
ii) Ferric iodide is very unstable but ferric chloride is not.
iii) Silver fluoride is fairly soluble in water while other silver halides are insoluble.
iv) The species [CuCl
4
]
2
exist but [Cul
4
]
2
does not.
Solution: i) Cu(l) salts under go disproportionation in aqueous solution
2
2Cu Cu Cu
+ +
+
Transition
Element_26
Address: MHV Plaza, Krishi FarmRoad, Kela Nagar Aligarh, Phone: 0571-6543330/1
ii)

I ion is a stronger reducing agent in comparision to Cl

ion.
3
Fe
+
is easily
reduced by

I ion
3 2
2
2Fe 2l 2Fe l
+ +
+ +
iii) The hydration energy of AgF is higher than its lattice energy. Hence it is soluble
in water. The hydration energy values of other halides are smaller than their
lattice energy values. Hence these halides are insoluble in water.
iv)

I is a stronger reducing agent than

Cl ion. It reduces
2
Cu
+
ion into Cu
+
ion.
Hence cupric iodide is converted into cuprous iodide. Thus the species
2
4
[Cul ]
does not exist.
Question6: Potassium pentacyanocobaltate (I I ) is diamagnetic in solid state but paramagnetic in
aqueous solution. Why?
Solution:
2 5
K [CO(CN) ] Potassium pentacyanocobaltate(II )
3 6
Co :d
+
configuration
As

CN is a strong ligand hence it forces the electrons to pair up. There are no
unpaired electrons left so diamagnetic. But in aq. solution the ligand is
2
H O which
is a weak one in comparision to

CN . There are 4 unpaired electronsand hence


paramagnetic.
Question7: FeSO
4
solution mixed with (NH
4
)
2
SO
4
solution (in the molar ration 1:1) gives the test
of Fe
2+
ion but CuSO
4
solution mixed with liquid NH
3
(the molar ration 1:4) does not
give the test of Cu
2+
. Explain why?
Solution:
4
FeSO does not from any complex with
4 2 4
(NH ) SO instead, they form a double salt
4
FeSO
4 2 4 2
(NH ) SO .6H O which dissociates completely into ions but
4
CuSO
combines with
3
NH to from the complex
3 4 4
[Cu(NH ) ]SO in which complex ion,
2
3 4
[Cu(NH ) ]
+
does not dissociate to give
2
Cu
+
ions.
Question8: Explain why a green solution of potassium manganate (VI)
2 4
K MnO turns purple
and a brown solid is precipitated when
2
CO is bubbled into the solution.
Solution:
2
CO in aqueous solution is acidic

2 2 2 3 3
CO H O H CO H HCO
+
+ +


In presence of H
+
,
2
4
MnO disproportionates to

4
MnO (purple, by oxidation) and
2
MnO (brown solid, by reduction)
2
4 4 2 2
VI VII IV
3MnO 4H 2MnO MnO 2H O
+
+ + +
+ + +
Question9: When mercury is oxidized with a limited amount of oxidizing agent (i.e., an excess of
Hg) then
I
Hg comounds are formed. I f there is an excess of oxidizing agent, then
I I
Hg compounds are formed.
2
Hg 2e Hg E 0.85V
+
+ =
2
2
Hg 2e 2Hg E 0.79V
+
+ =
Solution: The standard reduction potentials are so close that oxidizing agents like
3
HNO will
convert Hg to
2
Hg
+
rather than Hg(I) if the oxidizing agent is present in excess.
The reduction potential diagram shows that
2
2
Hg
+
is stable to disproportionation by a
small margin under standard conditions.
2 2
2
Hg Hg Hg E 0.06V
+ +
+ =

Transition
Element_27
Address: MHV Plaza, Krishi FarmRoad, Kela Nagar Aligarh, Phone: 0571-6543330/1
Question10: I n an atmosphere with industrial smog, Cu corrodes to a basic sulphate
( )
2 2 4
Cu OH SO cu and basic carbonate ( )
2 2 3
Cu OH CO . Propose a seriese of
chemical reaction to describe this corrosion
Solution:
2 3 2 2 2 4
2Cu H O SO O Cu (OH) SO + + +
2 3 2 2 2 3
2Cu H O CO O Cu (OH) CO + + +
Fortunately this corrosion product forms a tough adherent coating that protects the
underlyingmetal. Overall reaction is a combination of oxidation-reduction, acid-base
and precipitation reaction
Question11: Why is hydrochloric acid not used to acidify a permangnate solution is volumetric
estimation of Fe
2+
and C
2
O
4
2
.
Solution: This is because a part of the oxygen produced from KMnO
4
+HCl is used is oxidizing
HCl to Cl
2
4HCl +2(O) 2H
2
O +Cl
2
Question12: Why is copper sulphate pentahydrate coloured?
Solution: In the presence of H
2
O as ligand d-orbitals of Cu(II) ions split into two sets, one with
lower energy and the other with higher energy. From the white light falling on it, red
wavelength is absorbed or excitation of electron from lower to higher energy level. The
complimentary colour, viz blue is reflected.
Question13: Most of the transition metals do not displace hydrogen from dilute acids. Why?
Solution: This is because most of the transition metals have negative oxidation potentials.
Question14: K
2
[Pt Cl
6
] is well known compound whereas corresponding Ni compound is not
known. State a reason for it.
Solution: This is because Pt
4+
is more stable than Ni
4+
as the sum of four ionization energies of Pt
is less than that of Ni.
Question15: Why have the transition elements high enthalpy of hydration?
Solution: This is due to their small size and large nuclear charge. This is so because when we
move along any transition series the nuclear charge increases and size decreases.
13. OBJECTIVE ASSIGNMENTS
LEVEL I
1 Which of the following electronic configuration belong to transition elements?
(a) KL,
2 6 6 1
3s ,p d ,4s (b) KL,
2 6 10 2 3
3s p d ,4s p
(c) KL,
2 6 10 2 1
3s p d ,4s p (d) KLM,
2 6 10 2 1
4s p d ,5s p .
2. The no. of d-electrons present in
2
Fe
+
ions is
(a) 6 (b) 4
(c) 8 (d) 3.
3. Adams catalyst is
(a) Ft and PtO (b) Pt
(c) Pt and
2
PtO (d)
2
PtO and PtO
4. Which is the common oxidation state of the first transition series of elements?
(a) +2 (b) +6
(c) +8 (d) +4.
Transition
Element_28
Address: MHV Plaza, Krishi FarmRoad, Kela Nagar Aligarh, Phone: 0571-6543330/1
5. If orange-red colour is absorbed from white-light, the observed colour is
(a) Yellow (b) Orange
(c) Blue (d) Green.
6. An example of double salt is
(a) Bleaching powder (b)
4 6
K [Fe(CN) ]
(c) Hypo (d) Potash alum.
7.
2
Ti
+
is purple while
4
Ti
+
is colourless, because:
(a)
2
Ti
+
has
2
3d configuration
(b)
4
Ti
+
has
2
3d configuration
(c) There is no crystal field effect in
4
Ti
+
(d)
4
Ti
+
is very small cation when compared to
2
Ti
+
and hence does not absorb any radiation.
8.
4
FeSO solution gives brown colour ring in testing nitrates or nitrites. this is
(a)
2
2 5
[Fe(H O) NO]
+
(b)
2
2 5 2
[Fe(H O) NO ]
+
(c)
2
2 4 2
[Fe(H O) (NO) ]
+
(d)
2
2 4
[Fe(H O) NO]
+
.
9. Which shows maximum magnetic moment among the bivalent ions of the first transition series?
(a)
2
Fe
+
(b)
2
Co
+
(c)
2
Ni
+
(d)
2
Mn
+
10. Which is not true statement:
(a) ions of d-block elements are coloured due to dd transition
(b) ions of f-block elements are coloured due to ff transition
(c)
3 4
2 6 2 6
[Sc(H O) ] ,[Ti(H O) ]
+ +
are coloured complexes
(d) Cu
+
is colourless ion
11. Magnetic moment [Ag(CN)
2
]

is zero, the no. of unpaired electrons is


(a) Zero (b) One
(c) Two (d) Three
12. Which of the following is not a complex
(a) NiSO
4
.(NH
4
)
2
SO
4
.6H
2
O (b) [Co(NH
3
)
4
Cl
2
]Cl
(c) K
2
[Ni(CN)
4
] (d) [Pt(NH
3
)Cl
2
Br]Br
13. Which one is a monodentate ligand
(a) N C (b) NH
3
(c) H
2
O (d) All
14. The complex [Cr(H
2
O)
4
Br
2
]Cl gives the test for
(a) Br

(b) Cl

(c) Cr
3+
(d) Br

and Cl

both
15. In the reaction
4KCN +Fe(CN)
2
Product
The product formed can give the test of
(a) Fe
2+
(b) CN

(c) K
+
and [Fe(CN)
6
]
4
(d) CN

and [Fe(CN)
6
]
3-
16. Oxidation state of Cobalt in the complex [Co(NO
2
)
6
]
3
is
(a) +2 (b) +3
(c) +1 (d) +2 and +3
17. Coordination number of platinum in [Pt(NH
3
)
4
Cl
2
]
++
ion is
(a) 4 (b) 2
(c) 8 (d) 6
Transition
Element_29
Address: MHV Plaza, Krishi FarmRoad, Kela Nagar Aligarh, Phone: 0571-6543330/1
18. Which of the following is copper (I) compound
(a) [Cu(H
2
O)
4
]
2+
(b) [Cu(CN)
4
]
3
(c) [Cu(NH
3
)
4
]
2+
(d) All of these
19. Low spin complex is formed by
(a) sp
3
d
2
hybridisation (b) sp
3
d hybridisation
(c) d
2
sp
3
hybridisation (d) sp
3
hybridisation
20. Which of the following is a high spin complex
(a) [Co(NH
3
)
6
]
3+
(b) [Fe(CN)
6
]
4
(c) [Ni(CN)
4
]
2
(d) [FeF
6
]
3
LEVEL II
1. In
2 5
Na [Fe(CN) NO], sodium nitroprusside,
(a) oxidation state of Fe is +2 (b) this has NO
+
as ligand
(c) both correct (d) none is correct.
2. Stability of Cu
+
and Ag
+
halide complexes are in order:
(a) I >Br >Cl >F (b) F >Cl >Br >I
(c) Cl >F >I >Br (d) Br >I >Cl >F.
3. An acidic solution contains
2 2
Cu ,Pb
+ +
and
2
Zn
+
. If hydrogen sulphide gas is passed through this
solution, the precipitate will contain:
(a) CuS and ZnS (b) PbS and ZnS
(c) CuS and PbS (d) CuS, PbS and ZnS.
4. Magnetic moment of Cr(Z =24),
2
Mn (Z 25) andFe
+ +
= (Z =26) are x, y, z. They are in order:
(a) x <y <z (b) x =y <z
(c) z <x =y (d) x =y =z.
5. Select incorrect statement(s)
(a) ionization energies of 5d-elements are greater than those of 3d and 4d elements
(b) Cu(I) is diamagnetic while Cu(II) is paramagnetic
(c)
3
2 6
[Ti(H O) ]
+
is coloured while
3
2 6
[Sc(H O) ]
+
is colorless
(d) transition elements cannot form complexes.
6. AgCl and NaCl are colourless. NaBr and NaI are also colourless but AgBr and AgI are coloured.
This is due to :
(a) Ag
+
polarizes

Br and

I (b) Ag
+
has unpaired d-orbital
(c) Ag
+
depolarizes

Br and

I (d) None is correct.


7. The lowest degree of paramagnetism is shown by
(a)
4 2
MnSO .4H O (b)
4 2
FeSO .6H O
(c)
4 2
CuSO .5H O (d)
4 2
NiSO .6H O.
8. Which is correct statement?
(a) in less acidic solution
2 2 7
K Cr O and
2 2
H O give violet coloured diamagnetic

2
[CrO(O )(OH)] ion
(b) in alkaline
2 2 3 8
H O ,K CrO (with tetraperoxo species)
3
2 4
[Cr(O ) ] is formed
(c) in ammoniacal solution,
3 3 4
(NH ) CrO is formed
(d) all correct.
Transition
Element_30
Address: MHV Plaza, Krishi FarmRoad, Kela Nagar Aligarh, Phone: 0571-6543330/1
9. The structures of
4
Ni(CO) and
3 2 2
Ni(PPh ) Cl are
(a) square planar
(b) tetrahedral and square planar respectively
(c) tetrahedral
(d) square planar and tetrahedral respectively.
10. The hybridization states of the central atoms in the complexes
3
6
Fe(CN) ,
4
6
Fe(CN) , and
3
2 6
Co(NO ) are
(a)
2 3 3 4 2
d sp , sp andd sp respectively (b)
2 3 3 3 2
d sp , sp dandsp d respectively
(c)
2 3 3 2 2
d sp , spd anddsp respectively (d) all
2 3
d sp .
11. Among the following aquated metal ions, which has the highet degree of paramagnetism?
(a)
3
2 6
[Cr(H O) ]
+
(b)
2
2 6
[Fe(H O) ]
+
(c)
2
2 6
[Cu(H O) ]
+
(d)
2
2 2
[Zn(H O) ]
+
.
12. Which of the following is a high-spin (spin-free) complex?
(a)
3
3 6
[Co(NH ) ]
+
(b)
4
6
Fe(CN) ]
(c)
3
6
[CoF ] (d)
2
3 6
[Zn(NH ) ]
+
.
13. The formation of the complex ion
3
3 6
[Co(NH ) ]
+
involves the
3 2
spd hybridization of
3
Co
+
.
Therefore the complex ion should have
(a) an octahedral geometry (b) a tetrahedral geometry
(c) a square-planar geometry (d) a square-antiprismic geometry.
14. Which of the following will produce a white precipitate upon reacting with
3
AgNO ?
(a)
3 6 3
[Co(NH ) ]Cl (b)
3 3 3
[Co(NH ) Cl ]
(c)
2 2 2
K [Pt(en) Cl ] (d)
2 4
[Fe(en) ]Cl .
15. Which of the following is t-bonded organometallic compound?
(a) Ferrocene (b) Diethyl zinc
(c) Ethylmagnesium iodide (d) None of these.
16. Which of the following complexes produces three moles of silver chloride when its one mole is
treatedwith excess of silver nitrate
(a) [Cr(H
2
O)
3
Cl
3
] (b) [Cr(H
2
O)
4
Cl
2
]Cl
(c) [Cr(H
2
O)
5
Cl]Cl
2
(d) [Cr(H
2
O)
6
]Cl
3
17. Which is an example of coordination isomer
(a) [Co(NH
3
)
5
NO
2
]Cl
2
and [Cr(NH
3
)
5
ONO]Cl
2
(b) [Co(NH
3
)
6
][Cr(CN)
6
] and [Cr(NH
3
)
6
] [Co(CN)
6
]
(c) [Co(NH
3
)
5
SO
4
]Br and [Co(NH
3
)
5
Br]SO
4
(d) [Co(NH
3
)
4
(H
2
O)Cl]Cl
2
and [Co(NH
3
)
4
Cl
2
]Cl.H
2
O
18. The number of chloride ions which would be precipitated when the complex PtCl
4
.4NH
3
is treated
with silver nitrate is
(a) Four (b) One
(c) Three (d) Two
19. The number of ions per mole of the complex CoCl
3
.5NH
3
in aqueous solution will be
(a) Four (b) Nine
(c) Three (d) Two
20. In the complex [Co(en)
2
Cl
2
]Br, the coordination number and oxidation state of Cobalt are
(a) 6 and +3 (b) 3 and +3
(c) 4 and +2 (d) 6 and+1
Transition
Element_31
Address: MHV Plaza, Krishi FarmRoad, Kela Nagar Aligarh, Phone: 0571-6543330/1
Transition
Element_32
Address: MHV Plaza, Krishi FarmRoad, Kela Nagar Aligarh, Phone: 0571-6543330/1
14. SUBJECTIVE ASSIGNMENTS
LEVEL - I
1. Why Zn
+2
salts are white while Ni
2+
salts are blue?
2. Why the transition elements have high enthalpy of hydration?
3. Explain briefly why zinc and cadmium are soft metals?
4. Explain, the existence of
4
OsO in terms of trends in oxidastion states.
5. Explain the following
a) Mercury is a liquid
b)
2
TiO is white but
3
TiCl is violet
6. Determine the coordination number and the oxidation state of the transition metal ion in each of the
following complexes:
a)
4
6
[Fe(CN) ]
b)
3
6
[Fe(CN) ]
c)
3 2 2
[Pt(NH ) Cl ]
d)
2
4
[CuF ]
7. Yttrium with chlorine does not form YCl or
2
YCl but only
3
YCl . How does this agree with the trends
in stability of oxidation states?
8. Predict the spin only magnetic moment for:
a)
2
Fe
+
b)
7
Mn
+
c) Cu
+
d)
3
Ti
+
9. [NICl
4
]
2
is paramagnetic while [Ni(CO)
4
] is diamagnetic. Why?
10: [Ti(H
2
O
6
]
3+
is coloured whreas [Sc(H
2
O)
6
]
3+
is colourless, though it is a transition metal complex,
Explain the observation.
LEVEL - II
1. Write the formula ior each of the following complexes.
i) Ammonium heptafluorozirconate (IV)
ii) Tetraaquadichloroiron (III) ion.
iii) Dichlorobis(ethylenediamine) chromium (III) tetrachloropalladate(II)
iv) Dicyanobis(ethylenediamine) cobait (III) chlorate
v) Dichloro tetraammineplatinum (IV) ion
vi) Bromotriammineplatinum (II) nitrite
vii) Diaquatetrachlorochromate(III) ion
viii) Bis(cyclopentadienyl)iron (II)
ix) Hexaaquairon (II) sulphate
Transition
Element_33
Address: MHV Plaza, Krishi FarmRoad, Kela Nagar Aligarh, Phone: 0571-6543330/1
x) Potassium hexacyanoferrate (II)
xi) dichlorotetramminecobalt (III) ion
xii) Carbonylchlorobis (triphenylphoshine) iridium(l)
xiii) Tetraamminedichlorocobalt(III) hexacyanochromate(V)
xiv) -hydroxo-bis (pentammine chromium) (III)
xv) tri--carbonyl-bis (tricarbonyl iron)
2. Name the following complexes according to the IUPAC system of nomenclature
i)
3 4 2 3 2
[Co(NH ) (H O)Br](NO )
ii)
3 3
[Co(en) ]Cl
iii)
3 5 3 5 5
[(NH ) Cr OHCr(NH ) ]Cl
iv)
2 3 2
Ni(CO) (PPh )
v)
4 2 2
TiCl (Et O)
vi)
3
3 6
[Cr(NH ) ]
+
vii)
4 4
[Pt(Py) ][PtCl ]
viii)
2
4
[Zn(NCS) ]
ix)
3
4
[Cd(SCN) ]
x)
3 2 3
[Co(en)(NH ) ClBr]NO
3. Why
4
CuSO is blue while
4
ZnSO is white?
4. In
2
2 6
[Co(H O) ]
+
the observed magnetic moment is higher than the spin-only value. Explain the
reason for this in the space provided below.
5. Write down all possible isomers of
2 2 2
[Co(en) Cl ]NO and give their names.
6. The complex
2 3 2
[PdCl (PPh ) ] gives geometrical isomers and is found to be diamagnetic whereas an
analogous compound
2 3 2
[NiCl (PPh ) ] does not give any geometrical isomers and is found to be
paramagnetic. Explain these observations?
7. Each of the compounds
3 6 4 3 6 3 3 4 3
[Pt(NH ) ]Cl ,[Cr(NH ) ]Cl ,Co(NH ) .Cl and
2 6
K [PtCl ] has been
dissolved in water to make its 0.001 M solution. Rank then in order of their increasing conductivity in
solution.
8. Explain the following:
i) The complex
2
4
[CuCl ] exists, but
2
4
[Cul ] does not.
ii) Gold is not attacked by common acids but dissolves in aqua regia.
iii) Copper dissolves in aqueous KCN solution with the evolution of hydrogen.
iv) Zinc becomes dull in moist air.
9. The magnetic moment of
3
6
[Mn(CN) ] is 2.8 B.M and that of
2
4
[MnBr ] is 5.9 B.M. What are the
geometries of these complex ions?
10. How will you distinguish between the following isomer pairs
a) i) [CoBr(NH
3
)
5
]SO
4
and
ii) [Co(SO
4
) (NH
3
)
6
]Br
b) i) [Cr(H
2
O)
6
]Cl
3
and
ii) [CrCl(H
2
O)
6
]Cl
2
H
2
O
Transition
Element_34
Address: MHV Plaza, Krishi FarmRoad, Kela Nagar Aligarh, Phone: 0571-6543330/1
Level-I II
1. A coordination compound has the formula CoCl
3
.4NH
3
. It does not liberate ammonia but precipitates
chloride ions as silver chloride. Give the IUPAC name of the complex and write is structural formula.
2. In acidic solution silver (II) oxide first dissolves to produce
2
Ag
+
. This is followed by oxidation of
2
H O to
2
O and the reduction of
2
Ag
+
to Ag
+
. Write chemical equation for the dissolution and
redox reactions.
3. An aqueous solution of
3 3
CoCl .6NH is found to be a better conductor of electricity than an aqueous
solution of
3 3
CoCl .4NH . No ammonium ions are detected by the addition of
2 4
H SO to the solution
of either of these compounds. Can you explain the difference in the conductivities of two solutions?
4. Answer the following
i) Which one of
2
Fe
+
and F
3
e
+
ions is more paramagnetic and why?
ii) Which of the following ions are expected to coloured and why?
2 2 3 3 4
Fe ,Mn ,Cr ,Cu ,Se ,Ti
+ + + + + +
iii) Name the two elements of first transition series which have abnormal electronic configuration and
why?
5. Why is hydrochloric acid not used to acidify a permanganate solution in volumetric estimation of
2
Fe
+
and
2
2 4
C O .
6. Both
2
2 7
Cr O and

4
MnO solutions can be used to titrate
2
Fe
+
in acidic mediuim. Suppose you hae
available 0.1 M solutions of each. For a given sample of
2
Fe
+
solution, if a given titration requires
24.5 mL of 0.1 M
2
2 7
Cr O solution, how many mL of 0.1 M

4
MnO solution would have been
required if it had been used instead?
7. a) When Mn
2
(OH) is made by adding an alkali to a solution containing
2
Mn
+
ions, the precipitate
quickly darkens, and eventually goes black.What might be the chemical giving the black colour,
and how it is made ?
b) Dimercury (I) odide,
2 2
Hg I is a greenish colour and is precipitated if iodide ions are added to a
solution of dimercury (I) sulphate,
2 4
Hg SO . Likewise the red mercury (II) iodide,
2
HgI , is
precipitated from a solution of mercury (II) sulphate,
4
HgSO . However, both precipitates
dissolve in excess iodide solution. What might be the reason for this?
8. A solution containing 2.665g of
3 2
CrCl .6H O is passed through a cation exchanger. The chloride ions
obtained in solution react with
3
AgNO and give 2.87 g of AgCl. Determine the structure of the
compound.
9. Explain the following
i) A little acid is always added in the preparation of aqueous ferrous sulfate solution.
ii) Mercuric chloride and stannous chloride cannot exist as such if present together in an aqueous
solution.
iii) A ferrous salt turns brown in air.
iv) Copper hydroxide is soluble in ammonium hydroxide but not in sodium hydroxide.
10: Nickel can be determined by the precipitation of nickel dimethyl glyoximate.
a) What is the reaction?
b) A 1.502 g sample of steel yields 0.259 g of nickel dimethyl glyoximate. What is the per cent of Ni
is in the steel ? (Ni =59.0)
Transition
Element_35
Address: MHV Plaza, Krishi FarmRoad, Kela Nagar Aligarh, Phone: 0571-6543330/1
LEVEL-IV
1.
2
NiCl in the presence of dimethyl glyoxime (DMG) gives a complex which precipitates in the
presence of
4
NH OH, giving a bright red colour.
a) Draw its structure and show H-bonding
b) Give oxidation state of Ni and its hybridization
c) Predict whether it is paramagnetic or diamagnetic
2. Write the IPUAC nomenclature of the given complex along with its hybridization and structure.
2 3 4
K [Cr(NO)(NH )(CN) ], 1.73BM = .
3. Deduce the structures of
2
4
[NiCl ] and
2
4
[Ni(CN) ] considering the hybridization of the metal ion.
Calculate the magnetic moment (spin only) of the species.
4. A metal complex having composition Cr
3 4 2
(NH ) Cl Br has been isolated in two forms (A) and (B).
The form (A) reacts with
3
AgNO to give a white precipitate readily soluble in dilute aqueous
ammonia, whereas (B) givens a pale yellow precipitate soluble in concentrated ammonia. Write the
formula of (A) and (B) and state the hybridization of chromium in each. Calcualte their magnetic
moments (spin-only value).
5. a) Write the chemical reactions associated with the `brown ring test
b) Draw the structures of
3
3 6
[Co(NH ) ]
+
,
2
4
[Ni(CN) ] and
4
[Ni(CO) ]. Write the hybridization of
atomic orbitals of the transition metal in each case.
c) An aqueous blue coloured solution of a transition metal sulphate reacts with
2
H S in acidic
medium to give a black precipitate A, which in insoluble in warm aqueous solution of KOH. The
blue solution on treatment with KI in weakly acidic medium, turns yellow and produces a white
precipitate B. Identify the transition metal ion. Write the chemical reactions involved in the
formation of A and B.
6. A compound of vanadium has a magnetic moment of 1.73 BM. Work out the electronic configuration
of the vanadium ion in the compound.
15. ANSWERS
ANSWERS TO EXCERCISES
1. Cu
+
is a
10
d ion so that there is no dd transition and thus the ion is colourless. But
2
Cu O and
2
Cu S are coloured because of charge transfer of electrons from
2
O or
2
S to the vacant orbital of
Cu
+
.
2. Consider the two compounds
2
2 6
[Co(H O) ]
+
and
4
6
[Co(CN) ] each of which contains
2
Co
+
ion in
an octahedral field. Obviously the compound
2
2 6
[Co(H O) ]
+
is high spin having a magnetic moment
value of 4.6 BM whereas
4
6
[Co(CN) ] is low spin with a magnetic moment value of 1.9 BM. The
complex
4
2 6
[Fe(NO ) ] contains
2
Fe
+
which belongs to a
6
d system. As the magnetic moment of the
complex is zero, it indicates that all the six d-electrons are paired up. Hence the compound is low spin.
On the other hand, the magnetic moment of 5.94 BM for the complex
3
2 6
[Fe(H O) ]
+
corresponds to
five unpaired electrons in
3
Fe
+
. It indicates that there is no pairing of d-electrons of
3
Fe
+
ion, hence
the complex is high spin.
3: Here both the positive and negative part has the same metal. Procedure is same as earlier. The IUPAC
name is Tetrapyridlneplatinum(II) tetrachloroplatinate(II).
Transition
Element_36
Address: MHV Plaza, Krishi FarmRoad, Kela Nagar Aligarh, Phone: 0571-6543330/1
OBJECTIVE ASSIGNMENTS
LEVEL - I
1. (a) 2. (a) 3. (a) 4. (a) 5. (c)
6. (d) 7. (a) 8. (a) 9. (d) 10. (c)
11. (a) 12. (a) 13. (d) 14. (b) 15. (c)
16. (b) 17. (d) 18. (b) 19. (c) 20. (d)
LEVEL - II
1. (c) 2. (a) 3. (c) 4. (c) 5. (d)
6. (a) 7. (c) 8. (d) 9. (c) 10. (d)
11. (b) 12. (c) 13. (a) 14. (a) 15. (a)
16. (d) 17. (b) 18. (d) 19. (c) 20. (a)
SUBJECTIVE ASSIGNMENTS
LEVEL - I
1.
2
Zn
+
has completely filled d-orbitals
10
(3d ) while
2
Ni
+
has incompletely filled d-orbitals
8
(3d )
2. This is due to their small size and large nuclear charge. This is so because when we move along any
transition series the nuclear charge increase and size decreases.
3. Zn and Cd have electronic configuration
10 2
[Ar]3d 4s and
10 2
[Kr]4d 5s , respectively. Therefore, there
is no unpaired electron for metallic bonding. Thus, these metals are soft.
4. The oxidation number of osmium in
4
OsO is +8. The stability of higher oxidation states increases as
we go down the group of the transition metals. Osmium being in third transition series is, therefore,
stable in oxidation state +8 and exists as
4
OsO
5. a) No metallic bond formation
b) Ti(IV) had
0
d configuration
6. The coordination number and oxidation state of the metal ion in (a) to (d) are given below:
a) +6, +2 b) +6, +3
c) +4, +2 d) +4, +2
7. Stability of higher oxidation state increase as we go down a group.
8. Ion Electronic Number of Magnetic
Configuration unpaired electrons moment
a)
2
Fe
+ 6 0
[Ar]3d 4s 4 4.90
b)
7
Mn
+ 0 0
[Ar]3d 4s 0 0 BM
c) Cu
+ 10 0
[Ar]3d 4s 0 0 BM
d)
3
Ti
+ 1 0
[Ar]3d 4s 1 1.73 BM
9. In
4
[Ni(CO) ] Ni is in zero oxidation state whereas in
2
4
[NiCl ] , Ni is in +2 oxidation state. In the
presence of ligand CO, the unpaired electrons of Ni pair up but

Cl being a weak ligand is unable to


pair up the unpaired electrons.
10:
3
Ti
+
has some electron in the d-orbital
1
(3d ) which can absorb energy corresponding to yellow
wavelength and jump from lower energy level to higher energy level. But
3
Sc
+
has no electron in d-
orbital.
Transition
Element_37
Address: MHV Plaza, Krishi FarmRoad, Kela Nagar Aligarh, Phone: 0571-6543330/1
LEVEL II
1. i)
4 3 7
(NH ) [ZrF ]
ii)
2 2 4
FeCl (H O) ]
+
iii)
4 2 2 4
[Cr(en) Cl ] [PdCl ]
iv)
2 2 3
[Co(en) (CN) ]ClO
v)
2
3 4 2
[Pt(NH ) Cl ]
+
vi)
3 3 2
[Pt(NH ) Br]NO
vii)

4 2 2
[CrCl (H O) ]
viii)
5 5 2
[Fe(C H ) ]
ix)
2 6 4
[Fe(H O) ]SO
x)
3 6
K [Fe(CN) ]
xi)
2 3 4
[CoCl (NH ) ]
+
xii)
3 2
[lr(Ph P) (CO)Cl]
xiii)
3 4 2 6
[Co(NH ) Cl ][Cr(CN) ]
xiv)
5
3 5 3 5
[(NH ) Cr OHCr(NH ) ]
+
xv)
3 3 3
[(Co) Fe(CO) Fe(CO) ]
2. i) Bromoaquatetraamminecobalt (III)nitrate
ii) Tris(ethylenediamine)cobalt(III)chloride
iii) -hydroxobis(pentamminechromium(III))chloride
iv) Dicarbonylbis(triphenylphosphine)nickel(0)
v) Tetrachlorobis(diethylether)titanium(IV)
vi) Hexaamminechromium(III)ion
vii) Tetrapyridine platinium (II) tetrachloroplatinate(II)
viii) Tetrathiocyanato-N-zinc(II)
ix) Tetrathiocyanato-S-cadmium(I)
x) Diamminebromochloroethylenediaminecobalt(III) nitrate
3.
2
Cu
+
in
4
CuSO has
9 0
[Ar]3d 4s configuration and its electron can be promoted to the half filled d
orbital. Thus it can undergo d-d transition which absorbs mainlyin the red region of the visible light
and
4
CuSO appears blue in colour (blue is complementary colour of red). Because
2
Zn
+
in
4
ZnSO
has the configuration [A
10 0
r]3d 4s , the transition of electron from one d orbital to another is not
possible and no light is absorbed in the visible region of spectrum by
4
ZnSO and therefore, it appears
white.
4. The observed magnetic moment differs from that of the calculated spin only magnetic moment due to
the contribution of orbital motion of the electrons. The observed magnetic moment for
2
2 6
[Co(H O) ]
+
has contribution from the spin as well as orbital angular momentum and thusthe observed magnetic
moment is higher than the calculated spin-only magnetic moment.
5. The compound
2 2 2
[Co(en) Cl ]NO may have
2 2
[Co(en) (NO )Cl]Cl as an ionization isomer. This
ionization isomer in turn may give a linkage isomer
2 2
[Co(en) (NO )Cl]Cl . These three isomers would thus constitute what are known as structural
isomers. All three structural isomers can exist as cis- and trans- isomers, one such pair is given below:
The cis- form of each of the above three compounds would have an optical isomer or an enantiomer.
This would make a total of nine possible isomers for the given compound.
Transition
Element_38
Address: MHV Plaza, Krishi FarmRoad, Kela Nagar Aligarh, Phone: 0571-6543330/1
6. Since
2 3 2
[PdCl (PPh ) ] is a four coordinated complex and gives geometrical isomers, it must have
square planar geometry. Though
2 3 2
[NiCl (PPh ) ] is also four coordinated and belongs to a
8
d system
just like the first compound, yet it does not give geometrical isomers. This indicates a tetrahedral
geometry for the compound which will also be paramagnetic.
7. In aq. solution (0.001 M) the complexes will dissociate to give the ions.
4
3 6 4 3 6
[Pt(NH ) ]Cl [Pt(NH ) ] 4Cl
+
+

(5 ions)
3
3 6 3 3 6
[Cr(NH ) ]Cl [Cr(NH ) ] 3Cl
+
+

(4 ions)

3 4 3 3 4 2
Co(NH ) .Cl [Co(NH ) Cl ] Cl
+
+

(2 ions)
2
2 6 6
K [PtCl ] [PtCl ] 2K
+
+

(3 ions)
Thus, their conductivities (in solution) will increase with increasing number of ions liberated
so,
3 4 3 2 6 3 6 3 3 6 4
(leastconducting) (most conducting)
Co(NH ) .Cl K PtCl Cr(NH ) Cl Pt(NH ) Cl < < <
8. i)
11
Cu is reduced to
1
Cu by

I but not by

Cl .
ii) Theoxidation is aided by the complexation of the product gold (III) as

4
AuCl .
iii) The stability of
2
4
Cu(CN) ion is so great that the E value for oxidation is reduced to a point
where
2
H O can oxidise the copper.
iv) When zinc is exposed to moist air, the surface is affected with the formation of a film of basic
zinc carbonate on it. Due to this zinc becomes dull.
2 2 2 3 2
4Zn 3H O CO 2O ZnCO .3Zn(OH) + + +
9. For complex
3
6
[Mn(CN) ] , the number of unpaired electrons is calculated as,
2.8 n(n 2) n 2. = + =
3
6
[Mn(CN) ] has two unpaired electrons. Hence the geometry is octahedral with
2 3
d sp
hybridization. For complex
2
4
[MnBr ] , the number of unpaired electrons is calculated as
5.9 n(n 2) n 5 = + =
2
4
[MnBr ] , has 5 unpaired electrons. Hence the geometry is tetrahedral with
3
sp hybridization.
10. a) Isomer (i) gives white ppt of
4
BaSO with
2
BaCl whereas isomer (ii) does not form a ppt. At the
same time isomer (ii) gives a yellow precipitate of silver bromide with
3
AgNO but (i) does not.
b) The water molecule in Isomer (ii) lost easily on heating whereas the water molecule in isomer (i)
are not lost easily, being coordinated to the central atom.
LEVEL III
1. Remembering that co-ordination number of CO is 6 the formula of the complex will be
2 3 4
[CoCl (NH ) ]Cl . The name will betetraminedichlorocobalt (III) chloride.
2. Dissolution :
2
2
AgO 2H Ag H O
+ +
+ +
Redox :
2
2 2
4Ag 2H O 4Ag 4H O
+ + +
+ + +
3. As no ammonium ionis are detected in the solutions of these compounds, it indicates that ammonia
molecules are in the first coordination sphere of the metal ion in both the cases. The compound
3 3
CoCl .6NH can, therefore, be formulated as
3 6 3
[Co(NH ) ]Cl which may give four ions per mole in
solution as indicated by the conductivity of the solution. Since, the solution of
3 3
CoCl .4NH is less
conducting than that of
3 3
CoCl .6NH , it means it gives less than four ions per mole in solution. It will
Transition
Element_39
Address: MHV Plaza, Krishi FarmRoad, Kela Nagar Aligarh, Phone: 0571-6543330/1
be possible only if two of the chlorines are also present in the first coordination sphereand the
compound has the formula
3 4 2
[Co(NH ) Cl ]Cl .
4 i)
3
Fe
+
is more paramagnetic than
2
Fe
+
as
3
Fe
+
consists five unpaired electrons while
2
Fe
+
possesses four unpaired electrons.
ii) Any ion of transition elements which possesses unpaired d-electrons, i.e. dd transition is possible
shows a characteristic colour
0
(n1)d (or)
10
(n1)d configuration does not involve dd
transition and hence, is colourless.
2
Fe
+
,
2
Mn
+
and
3
Cr
+
are coloured, while
3
Cu , Se
+ +
and
4
Ti
+
are colourless.
iii) Chromium and copper. chromium attains
5 1
3d 4s configuration in which all the d-orbitals are
unpaired in order to get extra stability. Copper attains
10 1
3d 4s configuration in which all the d-
orbitals are paired in order to get extra-stability.
5. This is because a part of the oxygen produced from
4
KMnO +HCl is used in oxidizing HCl to
2
Cl
2 2
4HCl 2(O) 2H O 2Cl + +
6.
2 2 3 3
2 7 2
2 3
6 12
Cr O 14H 6Fe 2Cr 6Fe 7H O
+ + + +
+ +
+ +
+ + + +
2
2 7
Cr O 6e ] gain of

6e by two Cr atoms.
0.1 M
2 2
2 7 2 7
Cr O 0.6NCr O =
2 2 3
4 2
2 2 3
7
MnO 8H 5Fe Mn 5Fe 4H O
+ + + +
+ + +
+
+ + + +

4
MnO 5e
0.1 M

4 4
MnO 0.5N MnO
2
4
Fe 0.5N MnO
+

24.5 mL of 0.1 M
2
2 7 4
Cr O V mL od0.1M MnO
24.5 mL 0.6 N
2
2 7
Cr O V mL of 0.5 N

4
MnO
24.5 0.6
V 29.4mL
5

= =
7. a) The black colour is due to the manganese (IV) oxide,
2
MnO . It is made by the
2
Mn(OH) being
oxidized by oxygen in the air:
2 2
Mn(OH) MnO H O +
2 2
air black
1
MnO O MnO
2
+
b) It is due to formation of
2
4
HgI (a soluble complex) in both the cases with
2
HgI :
2
2 4
HgI 2I HgI +
But in
2 2
Hg I , first there is oxidation of Hg(I) to Hg(II) and then complex formation takes place;
it is by following disproportionation reaction:
2 2
2 4
0 1 2
Hg 4I HgI Hg
+
+ +
+ +
8. 143.45g of AgCl contains 35.45 g

Cl ions
2.87 g of AgCl will contain
35.45 2.87
143.45

=0.709 g

Cl ions
Transition
Element_40
Address: MHV Plaza, Krishi FarmRoad, Kela Nagar Aligarh, Phone: 0571-6543330/1
266.35 g
3 2
CrCl .6H O contains n 35.45 g of ionisable

Cl ions (where n =no. of

Cl ions outside
the coordination sphere).
Thus, 2.665 g
3 2
CrCl .6H O will contain
n 35.45 2.665
266.35

g
Also
n 35.45 2.665
0.709 n 2
266.35

= ~
Keeping in view the octahedral geometry of the complex, its structure may be written as
2 5 2 2
[CrCl(H O) ]Cl .H O
9. i) Ferrous sulfate is a salt of a weak base and a strong acid. Thus its hydrolysis occurs when it is
dissolved in water and solution becomes turbid due to formation of ferrous hydroxide
4 2 2 2 4
FeSO 2H O Fe(OH) H SO + +

Addition of a small amount of acid shifts the equilibrium towards left and prevents hydrolysis.
ii)
2
HgCl is an oxidizing agent while
2
SnCl is a reducing agent. When both are present, a redox
reaction occurs forming Hg and stannic chloride as final products.
2 2 2 2 4
SnCl 2HgCl Hg Cl SnCl + +
2 2 2 4
Hg Cl SnCl 2Hg SnCl + +
iii) A ferrous salt turns brown in air due to oxidation to ferric salt
iv)
2
Cu(OH) dissolves in
4
NH OH by forming a complex.
2 2 3 4 2 2
Cu(OH) 4NH OH [cu(NH ) ](OH) 4H O + +
2
Cu(OH) is insoluble in NaOH as no such complex is formed
10. a)
2
Ni
+
salts (in basic medium) on reaction with dimethyl glyoxime give cherry red ppt of nickel
dimethyl glyoximate:
2
NiCl 2 +
4
2NH OH +
C H
3
C
C C H
3
NOH
NOH
(in alcohol)
4 2
2NH Cl 2H O + +
N
C H
3
C H
3
N
O
Ni
O H
N
H O
O
N
C
C
CH
3
CH
3
(Cherry red ppt.)
b)
2
8 4 14 4
59 289
xg 0.259g
Ni (insteel) NiC N H O
+

Stioichiometrically:
x 0.259
59 289
=
x =0.05288 g pure Ni in 1.502 of steel sample.
Percentage of Ni =3.52%
Transition
Element_41
Address: MHV Plaza, Krishi FarmRoad, Kela Nagar Aligarh, Phone: 0571-6543330/1
LEVEL-IV
1. a) Structure of the complex is
C H
3
C N
C C H
3
N
O H O
O H O
N C CH
3
N C CH
3
Ni
Hydrogen bonding
Hydrogen bonding
b) The oxidation state of Ni in this compound is +2 and it shows
2
dsp hybridization.
c) Since the coordination number of Ni in this complex is 4, the configuration of
2
Ni
+
i.e.
3d 4s 4p
At first sight, gives the idea that the complex is paramagnetic with
3
sp hybridization as it has two
unpaired electrons. However the experiments show that the complex is diamagnetic i.e., it does
not have any unpaired electron. This is only possible when the 3d electrons rearrange themselves
against the Hunds rule i.e.,
3d 4s 4p
2
dsp hybridisation

This isalso in accordance with the fact that the ligand involve here is very strong.
2. IUPAC name of
2 3 4
K [Cr(NO)(NH )(CN) ] is potassium ammine tetra cyano nitroso chromate(I)
In it chromium is present as Cr(I), so
24 2 2 6 2 6 5 1
Cr 1s ,2s 2p ,3s 3p 3d ,4s =
2 2 6 2 6 5
Cr(I) Cr 1 ,2s 2p ,3s 3p 3d
+
= =
3d 4s 4p
For it 1.73BM =
n(n 2) = +
1.73 n(n 2) = + or
2
1.73 1.73 n 2n = +
Hence, n =1
On the basis of value of n =1, an unpaired electron is present is chromium (I) of this complex ion
Thus in excited state of Cr(I) of this complex ion
Thus in excited state of Cr(I) (i.e. in complete salt)
Transition
Element_42
Address: MHV Plaza, Krishi FarmRoad, Kela Nagar Aligarh, Phone: 0571-6543330/1
3d 4s 4p
2 3
d sp hybridisation

hence, in this complex Cr(I) shows


2 3
d sp -hybridisation, so structure of this complex is octahedral.
Cr
NO
NH
3
C
C C
C
N
N
N
N
2K
+
2
3. In
2
4
(NiCl )

Cl is a weak ligand which is unable to give pair of electron to


2
Ni
+
.
In
2
4
(NiCl ) oxidation state of Ni is +2
So,
2 2 2 6 2 6 8 0
Ni 1s ,2s 2p ,3s 3p 3d ,4s
+
=
Thus
3d 4s 4p
Co-ordination number of Ni is 4, so four vacant orbitals are required for the accommodation of four
pair of electrons of

Cl ions and there are one s and three p orbitals of valence sell.
Hence.
3d 4s 4p
3
sp hybridisation

The structure of
2
4
(NiCl ) is tetrahedral
Ni
Cl
Cl
Cl Cl
2
In it magnetic moment
n(n 2) 2(2 2) = + = +
(n 2 = (unpaired electron)) =2.82 B.M.
In
2
4
(Ni (CN) ) ,

CN is a strong ligand which is able to donate the pair of electron to


2
Ni
+
O. N. of
Ni in [(Ni
2
4
(CN) )] is also+2.
In
2
4
[Ni(CN) ) ],
2 2 2 6 2 6 8 0
Ni 1s ,2s 2p ,3s 3p 3d ,4s
+
=
3d 4s 4p
2
dsp hybridisation

3d 4s 4p
In ground state
In excitation
Hence the structure of
2
4
(Ni(CN) ) is square planer
Transition
Element_43
Address: MHV Plaza, Krishi FarmRoad, Kela Nagar Aligarh, Phone: 0571-6543330/1
Ni
C
C C
C
N
N
N
N
2+
2+
In
2
4
(Ni(CN) ) , no. of unpaired electron (n) =0
So, magnetic moment = n(n 2) 0 + =
4. A metal complex having composition Cr
3 4 2
(NH ) Cl Br have two forms A and B.
Form `A gives white ppt. with
3
AgNO , hence it must have chloride ion in form of non-complex ion
i.e. outside the complex sphere as
3 4
[Cr(NH ) ClBr]Cl

3 4 3 3 4 3
whiteppt Form`A'
[Cr(NH ) ClBr]Cl AgNO AgCl [Cr(NH ) ClBr] NO + + + +
These precipitates of AgCl are soluble in
4
NH OH due to formation of complex salt.
4 3 2 2
whiteppts Complexsalt
AgCl 2NH OH [Ag(NH ) Cl] 2H O + +
Similarly, form B gives pale yellow precipitate of AgBr which are sparingly soluble in
4
NH OH.
Hence form `B is
3 4 2
[Cr(NH ) Cl ]Br.

3 3 4 2 3 4 2 3
Paleyellowppt.
AgNO [(Cr(NH ) Cl )]Br AgBr [(Cr(NH ) Cl )] NO
+
+ + + +
4 3 2 2
Paleyellowppt.
AgBr 2NH OH [Ag(NH ) Br] 2H O + +
In both complexes, chromium is present as central ion and its oxidation number is +3. So in these
2 2 6 2 5 5 1
24
Cr 1s ,2s 2p ,3s 3p 3d ,4s =
3 2 2 6 2 5 3
Cr 1s ,2s 2p ,3s 3p 3d
+
=
Number of ligands are six and
3
Cr
+
shows
2 3
d sp hybridization in both complexes A and B
3d 4s 4p
2 3
d sp hybridisation

Hence in it, number of unpaired electrons are 3. So magnetic moment ( ) n(n 2) = +


(where n no. of unpaired electrons)
3(3 2) 15 3.872B.M. = + = =
5. a) Aqueous extract or soda extract is acidified with dil.
2 4
H SO , add freshly prepared
4
FeSO
solution and a few drops of conc.
2 4
H SO , appearance of brown ring confirms the presence of
nitrate radical.
3 2 4 4 3
2NaNO H SO NaHSO HNO + +
3 4 2 4 2 4 3 2
2HNO 6FeSO 3H SO 3Fe (SO ) 2NO 4H O + + + +
4 4
Brown ring (Ferrous nitro sulphate)
FeSO NO [Fe(NO)SO ] +
b) In
3
3 6
[Co(NH ) ]
+
cobalt is present as
3
Co
+
and its coordination number is six.
2 2 6 2 6 7 2
27
Coatom 1s , 2s 2p , 3s 3p 3d , 4s =
Transition
Element_44
Address: MHV Plaza, Krishi FarmRoad, Kela Nagar Aligarh, Phone: 0571-6543330/1
3 2 2 6 2 6 6
Co ion 1s , 2s 2p , 3s 3p 3d
+
=
3d 4s 4p
2 3
d sp hybridisation

3d 4s 4p
Co
3+
ion in
complex ion
Hence,
Co
NH
3
NH
3
NH
3
NH
3
H
3
N
H
3
N
3+
or Co
NH
3
NH
3
NH
3
NH
3
H
3
N
H
3
N
3+
Structure of
3
3
[Co(NH )]
+
is octahedral.
In
2
4
[Ni(CN) ] nickel is present as
2
Ni
+
ion and its coordination number is four.
2 2 6 2 6 8 2
28
Ni atom 1s , 2s 2p ,3s 3p 3d ,4s =
2 2 2 6 2 6 8
Ni ion 1s , 2s 2p ,3s 3p 3d
+
=
3d 4s 4p
2
dsp hybridisation

3d 4s 4p
Ni
2+
ion
Ni
2+
ion in
Complex ion
So structure of
2
4
[Ni(CN) ion is square planar which is represented as follows.
Ni
C
C C
C
N
N
N
N
2+
In
4
Ni(CO) , nickel is present as Ni atom i.e. its oxidation number is zero and coordination
number is four.
Ni atom =
2 2 6 2 6 8 2
1s ,2s 2p ,3s 3p 3d ,4s
3d 4s 4p
3
sp hybridisation

3d 4s 4p
Ni
2+
ion
Ni in complex
So structure of
4
Ni(CO) is tetrahedral which is represented as shown:
Transition
Element_45
Address: MHV Plaza, Krishi FarmRoad, Kela Nagar Aligarh, Phone: 0571-6543330/1
Ni
OC
CO
OC CO
c) The transitional metal is
2
Cu
+
.
The compound is
4 2
CuSO .5H O. It dissolves in water to give blue coloured solution due to
presence of
2 9
Cu (d
+
-configuration). On passing
2
H S in acidic medium in such solution, the
black precipitate of CuS is obtained which is not soluble in aq. KOH (warm) solution.
Acidic
4 2 2 4 medium
Blackppt.
(Insolublein aq. KOH)
CuSO H S CuS H SO + + +
On addition of KI solution in aqueous solution of
4
CuSO , it produces yellow coloured solution
of
2
CuI which is decomposed in white ppt. of
2 2
Cu I and liberate
2
I .
4 2 2 4
CuSO 2KI CuI K SO + +
2 2 2 2
whiteppt.
2CuI Cu I I + |
6. As we show that Magnetic moment ( ) n(n 2) BM = +
where n number of unpaired electrons 1.73 = BM for vanadium ion
1.73 BM n(n 2) = + . So
2
(1.73) n(n 2) = +
3.0 =
2
n 2n + or
2
n 2n 3 0 + =
2
n 3n n 3 0 + =
n(n 3) 1(n 3) 0 + + =
(n1)(n 3) 0 + =
Correct value of n =1. Thus no. of unpaired electrons in vanadium ion =1
2 2 6 2 6 3 2
23
V 1s ,2s 2p ,3s 3p 3d ,4s =
So in vanadium (IV) an unpaired electron is present.
4 2 2 6 2 6 1
V 1s ,2s 2p ,3s 3p 3d
+
=

S-ar putea să vă placă și